إعـــــــلان

تقليص
لا يوجد إعلان حتى الآن.

مختبرات مستوى خامس فقط

تقليص
X
 
  • تصفية - فلترة
  • الوقت
  • عرض
إلغاء تحديد الكل
مشاركات جديدة

  • مختبرات مستوى خامس فقط

    1:Advanced CliniCal Chemistry
    2: Virlogy
    3:Clinical Histotechnology
    4:Clinical Blood Banking &Transfusion Sciences
    5:Biostatistics &Lab Management

    انا مستوى خامس و ما لقيت اسئلة

    و محتاااااااااااجة اسئلة هادي المواد بلييييز ساعدوني

    اختباراتي دي الاسبوعين

    اللي عنده بليييز لايبخل
    :sm202::sm177:
    الملفات المرفقة
    التعديل الأخير تم بواسطة مجنونة بس حنونه; الساعة 09-01-2012, 01:16 PM.

  • #2
    11 مشاهدة و ما في احد عنده :sm184:

    تعليق


    • #3
      اسئلة فايرولوجي
      الملفات المرفقة

      تعليق


      • #4
        يعطييك ربي العافية ....يسلمووا اذا عندك كمان اسئلة للمواد البباقية ابغاهن ضرروري ووشكرا .......

        تعليق


        • #5
          هذه الامتحانات من إخوة و لا أنسبها لنفسي و شكرا لهم جميعا و أدعوا لنا جميعا

          ربنا يوفقنا جميعا و لكني لست متأكدة من اجاباتهم لكني لا أدري المستوى الخامس لكن المواد نفسها موجودة و سأضيف لكم بعد قليل الملخصات العشرة لإجتياز الهيئة لأخونا شادي جزاه الله خيرا هو و من ساعده فيها انتظروني و شكرا لكم دعواتكم لنا و لكم جميعا.

          I- الموضوع: اختبـــار الهيئـه 2012:
          09-01-2012, 02:38 PM #2
          من موقع مختبرات العرب
          http://www.arabslab.com/vb/showthread.php?t=28931
          هذه أسئلة و لكن الإجابات غير مؤكدة لأنها إجابة طالب :
          1. 10-01-2012, 08:02 PM #9
          سعودي 2000
          عضو نشيط


          1-Taenia saginata diagnostic by
          a. Gravid segment
          b. Ova in stool c. Cyst
          d. Trophozoite
          e. Larva in stool.


          2-Fasciola live in
          a. Liver and bile duct b. Pancrease
          c. Stomach
          d. Kidney



          3-Gram positive bacteria stained
          a. Dark purple b. Yellow
          c. Green
          d. Red

          4-Hydrogene peroxide used in catalase reaction concentration is
          a. 3% b. 40%
          c. 100%
          d. 50%
          5-Haemophilus influenza grow well in
          a. Chocolate agar b. MacConky
          c. Blood agar
          d. Manitol salt agar

          6-Hyperglycemic hormones
          a. Amylase
          b. Insulin
          c. Lipase
          d. Glucagon e. Trypsin

          7-In the lab. An individual must wear
          a. Wearing lab coat
          b. Eye glasses
          c. Gloves
          d. ALL

          8-Life Spain of platelets in bags stored at R.T.
          a. 35 day
          b. 41 day
          c. 3 months
          5 day

          9-One of the following tests not performed in chemical section in the lab.
          a. Liver enzymes
          b. Hormones
          c. Bowne marrow aspiration Creatinin


          10-Rice watery stool can caused by
          a. Salmonella
          b. Vibro cholera c. Shigella
          d. E.col
          البيئه اللي تنمو عليها


          11-Which of the following leukocyte responsible for parasitic & allergic reaction
          a. Lymphocyte
          b. Neutrophile
          c. Monocyte
          d. Basophile
          e. Eosinophile
          12--which of the following produce by staph.auerus & not produced by other staph
          a. DNA produce
          b. Coagulase c. Catalase
          Ureaase


          13-In emergency case which of the following test do for glucose
          a. Fast blood sugar
          b. Random blood sugar c. Oral blood sugar tolerance
          Post prandial blood sugar

          14-Which of the following immunoglubin have four molecules?
          a. IgG
          b. IgE
          c. IgA
          IgM

          15-The cause of chaga's disease:
          a. Trypanosoma gambiense
          b. Trypansoma rhodesiense
          c. Trypansoma cruzi Leishmania braziliense

          16-Which if the following leukocyte normally not found in blood film?
          a. Basophile b. Monocyte
          c. Lymphocyte
          d. Neutrophils
          Eosinoph
          17-HbA1C for
          a. 1-2 Weeks
          b. 2-4 Weeks
          c. 4-6 Weeks
          d. 6- 8 Weeks

          18-All are true about platelet except
          a. Can save for 7 days when added special fluid b. Can save for 5 days when added special fluid
          c. Defect cause prolonged bleeding time
          d. Normal 150,00 – 450,000

          19-Antibodies contain four IG
          a. Ig M
          b. Ig A
          c. Ig D
          d. IgE
          IgG

          20-Platelet normal range:-
          a. 100×10(12)
          b. 20×10 (9)
          c. 100×10(9)
          d. 1000×10(9)


          21-The gram stains use for:
          a. Mycobacteruim
          b. Borrela
          c. Dipheteria
          E.coli
          22-Hypocalcaemia in patient occur if the level of calcium is:
          a. 16
          b. 12
          c. 10
          d. 7.5
          23-Most common method used to detect HBV:
          a. Microscope
          b. Flamcytometer
          c. ELIZA d. PCR


          24-Lenses near the slide in light microscope:
          a. Objective=العدسة الشيئية b. Stage
          c. Eye lenses=العدسة العينية
          d. Arm


          25-Gas gangrene cause by :
          a. Clostridium botulism
          b. Clostridium tetani
          c. Clostridium prefringers


          26-Storage temperature of platelets
          a. 37 C
          b. 35 – 40
          c. 22 – 27 d. -4

          20-22 جايبين هالاختيار
          27-The cell wall of gram(-) contain :
          a. toxic lipo poly saccharide b. Peptedoglycan only
          c. Techoic acid only
          d. Celleluse and pectin


          28-The triple granule, which is the most aboundent in leucocyte, is:
          a. Netrophil b. Eosinophil
          c. Basophil
          d. Lymphocyte
          29-The following parasite will not take the gimsa stain:
          a. leishmanai.
          b. Trypanosoma
          c. Schistosoma
          d. Malaria Filaria

          30-When the micro invades, the body B-cell will secret and it will transfer to :
          a. Memory cell
          b. Plasma cell c. Phygocytic cell
          d. Cytotoxic

          31-Food poisoning caused by
          a. Closteridium tetani
          b. E.coli
          c. Cambylobacter
          d. Closteridium perfringens
          32-Heprenized syringe use to collect __________ sample
          a. Synovial
          b. CSF
          c. Venous =الوريد d. Arterial


          33-Test performs in blood banking lab.
          a. HBVc Ab
          b. HBVs Ag
          c. HBVs Ab
          d. ALL
          34-Largest cells in blood film
          a. Monocyte b. Lymphocyte
          c. Neutrophil
          Eosinophil

          35-Monospot test is done to diagnose
          a. Infectious mononucleosis b. Scarlet fever
          c. Reumatic fever
          d. AIDs

          36-Scotch-tap test used for
          a. Ascaris lumbricoides
          b. Ancylostoma duodenal
          c. Trichuris tricura
          d. Taenia saginata
          e. Enterobius vermicularis
          37-To fixation of thin blood film we can used
          a. Aceton
          b. Acitic acid
          c. Acid –alchol
          Not fixe




          38-Zile-Neelsen stain for staining
          a. Mycobacteria b. E.coli
          c. Corynebacterium diphteria
          d. Salmonella

          39-Normal person,with healthy condition but Ketone bodies appear in his Urine due to
          a. Drugs
          b. Prolonged fast c. None restricted food
          d. UTI

          II-هذا من شخص آخر يسامحني لا أذكر اسمه:
          1. نموذج 1

          1) The definitive host is an animal:
          1-Entamoeba histolitica
          2-Toxoplasma Gondii
          3-Trypanosoma cruzi
          4-E.coli
          5-Giardia lamblia
          2) One is not found (involve) in colorimetric:
          1- Cuvett
          2-light source
          3- Photo sensor and analyzer
          4-fule source
          5-Filter
          3) The smallest molecular weight Ig is:
          1-IgG
          2-IgM
          3-IgD
          4-IgE
          5-IgA
          4) Bacteria can cause pathogenesis to human by:
          1-Capsullar
          2-Secret enzymes
          3-Endogenous toxins
          4-Exogenous toxins
          5-All of the above
          5) Neutrophil count is high in:
          1-Acute bacterial infection
          2- Iron deficiency anemia
          3-Megaloblastic anemia
          6) One is not correctly paired:
          1. α- cells→ glucagon
          2. β- cells → insulin
          3. Parathyroid → calcium
          4. Corpus luteum → Progesterone
          5. Estrogen → seminiferous tubules
          7) Function of pili of the bacteria:
          1- Attachment to the host tissue
          2- Movement
          3- Reproduction (multiplication)
          4- Engulf of food
          5- All of the above
          8) Hb A2 is consisting of:
          1- 3 ά chains and 2 γ chains
          2- 2 ά chains and 2 β chains
          3- 2 ά chains and 2 δ chains
          4- 2 ά chains and 3 δ chains
          5-3 ά chains and 2δ chains
          9) The main (most) Hb found in adult is:
          6. Hb A
          7. Hb A2
          8. Hb F
          10) Blood medium commonly used in blood bank:
          9. Citrate Phosphate Dextrose adenine (CPD-A)
          10. EDTA anticoagulant
          11. Heparin anticoagulant
          12. Saline alanin glucose maltose (SAGM)
          11) Indirect anti-antibody test used to detect:
          13. Sensitized RBCs in patient blood
          14. IgG
          15. IgM
          16. Sensitized antibody in patient serum
          17. Non of the above
          12) Direct anti-antibody test used to detect:
          1- Sensitized RBCs in patient blood
          18. IgG
          19. IgM
          20. Sensitized antibody in patient serum
          21. Non of the above
          13) Anti-Human Immunoglubin is:
          22. Coombs reagent
          23. Anti-Ab
          24. IgG
          25. All of the above
          26. Non of the above
          14) Rh-ve patient means he dose not have:
          27. E antigen
          28. D antigen
          29. B antigen
          30. C antigen
          31. Non of the above
          15) Detection of malaria parasite is by:
          1- Thick blood film
          16) One is always non motile gram –ve rods:
          1-Haemophilus Influenza
          2-Shigella
          3- E.coli
          4- Salmonella
          5- Borditella Pertusis
          17) One is motile gram –ve rods:
          1- Haemophilus Influenza
          2- Bacillus anthracis
          3- Pseudomonas aerogenes
          4- Vibrio cholerae
          5- Yersinia Pestis
          18) One is gram +ve oval (cocci):
          1- Bacillus anthracis
          2- Meningococci
          3- Pneumococci
          19) A child diagnosis show scarlet fever the cause is:
          1- Streptococcus Pyogenes
          2- Staphylococci aureus
          20) Xylose lysine Deoxycholate (XLD) is:
          1- Selective media
          2- Simple media
          3- Differential media
          4- Enriched media
          5- Enrichment
          21) Function of Thyroxin is:
          1- Increase Oogenesis
          2- Stimulate contraction of uterus
          3- Increase basal metabolic
          22) Increase in Thyroxin Stimulating Hormone (TSH) causes:
          1- Cushing disease
          2- Gigantism
          2- Exophthalamic goiter
          3- Hypoglycemia
          23) Function of Estrogen:
          1- Increase Oogenesis
          2- Stimulate contraction of uterus
          3- Increase basal metabolic
          24) Infective stage of Entrobius Vermicularis is:
          1- Larva
          2- Cercaria
          3- Egg
          4- Cyst
          5- Metacercaria
          25) Infective stage of Taenia:
          1- Larva
          2- Cercaria
          3- Embryonated egg
          4- Cyst
          5- Metacercaria
          26) Autoinfection of human can caused in case of:
          1- Entrobius Vermicularis
          2- Taenia Sp.
          3- Anclystoma Dudenal
          4- Ascaris Lumbercoidis
          27) The leukocyte that involve in Adaptive and Acquired immunity:
          1- Lymphocyte
          2- Neutrophil
          3- Monocyte
          4- Basophile
          5- Eosinophil
          28) The atomic mass of an atom is the number of:
          1- Protons and neutrons
          2- Protons only
          3- Neutrons only
          4- Electrons
          5- Non of the above
          29) In case of obstructive jaundice one is commonlynot found:
          1- Total bilirubin is elevated
          2- High bilirubin in urine
          3- Direct bilirubin is within the normal
          4- Dark color of urine
          5- Clay color of stool


          يتبع
          التعديل الأخير تم بواسطة alsadi ; 07-21-2008 الساعة 08:50 PM
          ________________________________________
          2. 07-21-2008, 08:29 PM #4
          30)In case ofHemolytic jaundice one is not likely to be found:
          1- Total bilirubin is elevated
          2- High bilirubin in urine
          3- No chaing in color of urine
          4- Direct bilirubin is within the normal
          5- Normal color of stool
          31) One is not true about acid:
          1- Proton donor
          2- Turn litmus paper to blue
          3- Sour taste
          4- React with alkaline to give water and salt
          5- Non of the above
          32) Pentose phosphate metabolism cycle isimportant to the cell because it give:
          1- Acetyl CoA
          2- ADP
          3- ATP
          4- NADH
          5- NADPH
          33) Hydrolysis of sucrose result:
          1- Only glucose
          2- Galactose
          3- Fructose + glucose
          4- Glucose + Glucose
          5- Maltose + Glucose
          34) One is not a Romanowsky stain:
          1- Fields stain
          2- Gram stain
          3- Geimsa stain
          4- Leishman stain
          35) Romanowsky stain consists of:
          1- Eosin + Alkaline methylene blue
          2- Eosin only
          3- Methylene blue only
          4- Indian Ink
          36) Deionization of impure water means:
          1- Boiling
          2- Filtration
          3- Exchange of protons and electrons
          37) Iodine stain of Entamoeba cyst used to detect(diagnose):
          1- Chromatoidal bars
          2- Cell wall
          3- Nuclei
          38) Hb A consists of:
          1- 2 ά chains and 2 δ chains
          2- 3 ά chains and 2 β chains
          3- 2 ά chains and 2 β chains
          4- 2 ά chains and 3 β chains
          5- 2 ά chains and 2 γ chains
          39) Normal range of leukocyte is:
          1- 4-11 X 10^9
          40) Diagnostic stage of Ascaris lumbericoidis:
          1- Finding of a typical fertilized egg in stool
          41) Life span of red blood cells in the circulation(body):
          1- 80 days
          2- 120 day
          3- 130 day
          4- 20 day
          5- 7 days
          42) Increase in growth hormone causes:
          1- Cushing disease
          2- Gigantism
          3- Exophthalamic goiter
          4- Hypoglycemia
          43) If you see this sign (symbol) in the lab it means:
          1- Flammable
          2- Corrosive
          3- Oxidizing
          4- Explosive
          5- Toxic
          44) Reticulocyte is immature:
          1- RBC
          2- WBC
          3- Platelet
          45) Best sterilization of Nutrient media done by:
          1- Hot air oven
          2- Autoclaving
          46) Most common method (technique) used to detect hormone amount in the laboratories:
          1- Spectrophotometry
          2- Enzyme Linked Immuno Surbant Assay (ELIZA)
          47) Test should be kept away from light:
          1- Serum bilirubin
          48) Immunoglobulin pregnancy test related to:
          1- IgG
          2- IgM
          3- Human chorionic Gonadotropin
          49) Leukocyte responsible for cellular immunity:
          1- T-lymphocyte
          2- B- lymphocyte
          3- Monocyte
          4- Basophile
          5- Eosinophil
          يتبع
          ________________________________________
          07-21-2008, 08:32 PM #5
          50) Buffer solution is:
          1- Chaing color when chaing pH
          2- Resist acidic pH
          3- Resist alkaline pH
          4- Weak acid + weak base
          5- Strong acid + weak base
          51) An indicator is:
          1- Chaing color with chaing pH
          2- Resist acidic pH
          3- Resist alkaline pH
          4- Weak acid + weak base
          5- Strong acid + weak base
          52) Target of prolactin in female:
          1- Ovary
          2- Mammary glands
          53) Parasite can be detected in vaginal and urethraldischarge:
          1- Trichomonas Vaginalis
          54) Org. (bacteria) arranged in Chinese litters:
          1- Corynebacteria Diphtheria
          2- Bacillus anthraces
          55) Malaria infection transmitted by:
          1- Male anopheles mosquito
          2- Female anopheles mosquito
          56) Heparin is:
          1- Protein
          2- Enzyme
          3- Polysaccharide
          4- Oligosaccharide
          5- Non of the above
          57) One stage prothrombin time used to detect (diagnose) disorders in:
          1- Extrinsic coagulation pathway (factors)
          2- Intrinsic coagulation pathway (factors)
          58) Thrombin time is:
          1- Extrinsic coagulation pathway (factors)
          2- Intrinsic coagulation pathway (factors)
          3- The conversion of prothrombin to fibrinogen in addition of thrombin
          59) Confirmatory test of streptococcus pneumonia:
          1- Catalase
          2- Optochine disc
          3- Coagulase
          4- Bile insolubility
          5- Bacitracin
          60) Most sever (serious) malaria infection caused by:
          1- Plasmodium Falcibarum
          2- Plasmodium Malaria
          3- Plasmodium Ovale
          4- Plasmodium Vivax
          5- All of the above
          61) Parasite can cause fetal cerebral infection:
          1- Taenia
          2- Toxoplasma gondii
          62) Gram +ve cocci arranged in groups (clusters):
          1- Streptococci
          2- Staphylococci
          63) Non motile protozoa:
          1- E.coli
          2- Entamoeba Histolytica
          3- Trypanosoma cruzi
          4- Giardia lamblia
          5- Trichomonas vaginalis
          64) One is always oxidase +ve
          1- Haemophilus Influenza
          2- Pseudomonas aurogenase
          65) Leukocyte responsible for response to parasitic and allergic infection:
          1- Lymphocyte
          2- Nutrophile
          3- Monocyte
          4- Basophile
          5- Eosinophil
          يتبع

          ________________________________________
          نموذج 2
          1-Blood of Microfilaria infection diagnostic with:
          Wuchereria bancrofti
          2-Oocyst benefited to diagnose:
          E.histolytica
          E.coli
          Toxoplasma gondii
          Giaradia lamblia
          Typanosoma cruzi
          3-intracellular parasite:
          E.coli
          Toxoplasma gondii
          Giaradia lamblia
          Typanosoma cruzi
          Trichomonas vaginalis
          4- An animal is a definitive host:
          E.histolytica
          E.coli
          Toxoplasma gondii
          Giaradia lamblia
          Typanosoma cruzi
          5-cosnider ketose:
          Glucose
          Fructose
          Mannose
          6- non motile protozoa:
          E.histolytica
          E.coli
          Toxoplasma gondii
          Giaradia lamblia
          Typanosoma cruzi
          7-cutaneous leishmaniasis transmitted by:
          Phlebotomus sand fly

          8-malaria transmitted by :
          Female anopheline mospuito
          Male anopheline mospuito
          9-shistosoma hematobium diagnosis:
          Egg in urine with terminal spine
          10-not find in the flam photometry :
          Burner
          Filter
          Fuel source
          Cuvete for sample
          Photo
          11-low effective sterilization with:
          Ethyl alcohol 70%
          Methyl alcohol
          chloroform
          5%phenol
          5% cresol
          12-prolonged fast cause:
          Hyperglycemia
          Keton in urine
          13-One is not correctly paired:
          α- cells →glucagon
          β- cells → insulin
          Estrogen → seminiferous tubules
          Corpus luteum → Progesterone
          Parathyroid → calcium
          14- bacteria motile gram-rods:
          Shigella
          Bordetela pertusis
          Pseudomonas aeruginoea
          Yersenia pestis
          15-presence one from this substance in urine detect abnormal codition;
          Calcium
          Phosphates
          Glucose
          Urates
          16-hypersecretion of cortisol cuase:
          Cushing disease
          17-in cardiac infraction one of the measurements is not benefited:
          Creatinine
          18-elevated in parasitic infection and allergy :
          IgE
          19- with romanowsky stain ,2-5 lobes and give violet or pinkish granules:
          Nutrophile
          Eiosinophile
          Basophile
          Monocyte

          يتبع
          ________________________________________
          07-21-2008, 08:40 PM #7
          20-lense near the slid in light microscope :
          Objective
          Eye lenses
          21-one of the following has highest conc. Of cholesterol:
          Chylomicron
          LDL
          HDL
          VLDL
          Apolipoprtein
          22-hypersecretion of insulin cause:
          Hypoglycemia
          23-hyperglycemia hormone
          Glucagon
          24- B-lymphocyte after attack foreign bodies ,called
          Plasma cell
          25- technique used for detect Hormon amount:
          ELIZA
          26-does not sterile with hot air oven:
          Dry glassware
          Oil
          Powder
          Waxes
          Rubber gloves

          27-lymphocyte is elevated in :
          Viral infection
          Acute bacterial infection
          Iron deficiency anemia
          Megaloblastic anemia
          non of the above
          28- The smallest molecular weight Ig is:
          1-IgG
          2-IgM
          3-IgD
          4-IgE
          5-IgA
          29-gram-v bacteria colonies color :
          Dark purple
          Pale to dark red
          Orange
          30- bacteria need dark field to detect
          T.pallidium
          31-Autoclave sterilize in temp:
          121ْc-20min
          32-strict aerobe bacteria
          Mycobacterium
          33-citrate test assis :
          Mycobacterias
          Staph
          Strept
          Colistridia sp
          34-nutrent agar:
          Basic media
          35- hypoparathyrodism hormone cause:
          Elevated ca level
          Decrease ca level
          36-use O cell tube on lab for:
          As control
          To detect Ag
          To detect Ab
          37-all cause false –v ABO blood grouping except
          Too short incubation
          38-prothrompin time detect:
          Extrinsic pathway of coagulation
          39-alkali skin burn treatment by neutralization with:
          Sodium bicarbonate powder
          Boric acid
          Acetic acid 1%
          Cold water
          Hot water

          يتبع
          ________________________________________
          07-21-2008, 08:47 PM #9
          40-acid skin burn treatment by neutralization with:
          Sodium bicarbonate powder
          Boric acid
          Acetic acid 1%
          Cold water
          Hot water
          41-store RBC at -80ْc:
          1year
          1month
          4 year
          2 year
          6year
          42-deffenetial test for:
          Leukocyte
          43- disease caused by pyogen stereptococcus:
          Scarlet fever.
          44- you do not take this stage for tratment small cut In emergency :
          Clean with soap and water
          Do pressure with piece
          Immediately rinse mouth well and water
          Cover it with water dressing
          Sterilize

          45- org cause bloody in stool:
          Shigella sp.
          46-sensitized Ab in serum detect by:
          Indirect antiglublin test
          47-thrombin time measure:
          Convert fibrinogen to fibrin with activate of thrombin
          48- in presence of clotting defect one of this not measure:
          Leucocyte count
          49-in Rh-v person , this Ag not found :
          D-antigen
          50-Z.N stain use for diagnose :
          Mycobateria
          51-media use for differentiate between L.F and N.L.F:
          Maconky media
          52-bleeding time test detect the abnormality in :
          Defect in vessels and platelets
          53- Immature RBC that is normally presence in blood film:
          Erythrocyte
          54- erythrocyte is immature cell of:
          RBC
          55-S hemoglobin is the same defect of thalassemia but the different is :
          Long B chain
          Glutamic acid on B chain is substituted with valine
          Glutamic acid on a chain is substitutes with valine
          56- hormone that cause replication of RBC:
          Erythropoietin
          57- use for clean slide:
          Ethyle alcohol
          Methyle alcohol
          58-oxytocine hormone function:
          Contribtion of Uterus
          59-regulation of calcium level in serum by:
          Calcitonine hormone only
          Parathyroid hormone only
          Calcionine with parathyroid hormone
          Vitamin D
          PTH+Vitamin D+calcitonin
          60-the defect of thalassemia occur in:
          Cycle of heme
          Glubin chain
          61-Normal range of WBCs:
          4000_11000
          62-stain use for tuberculosis:
          Lewis Jensen stain
          63- pyrimidin base:
          Adenine
          Guanine
          Thymine
          Uric acid
          Urea
          64- one use for measure abnormality in kidney :
          Creatinine
          65- not romanowsky stain :
          Gram stain
          66-does not stimulate with slight hemolysis:
          Cholesterol
          67-one of them use for susceptible abnormality kidney:
          Creatinin
          68- in O+v person
          No presence of Ag
          No presence of Ab
          69-one is stimulate on serum calcium level:
          Prolonged application of tourniquet on arm
          70-Eiosinphil is high in :
          Parasitic infection and allergic reaction.
          .
          ________________________________________
          07-21-2008, 08:56 PM #10
          نموذج 3


          0. the infective stage of animal parasite

          o Toxoplasma gondii

          2. the (u) unit used to evaluation of :

          o Protein in serum
          o Hemoglobin
          o Hormone assay
          o Enzymatic activity

          7. the following is ingredient of culture medium:

          o Meat extract
          o Minral salts
          o Agar
          o Peptone
          o Non of the above
          o All of the above

          14. a semen specimen, contain sugar source of the spermatozoa energy is:

          o Glucose
          o Mannose
          o Glactose
          o Fructose
          o Sucrose
          5.CPD_A anticoagulant used to store blood for:

          o 35 days
          o 45 days
          o 5 days
          o 15 days
          o Non of the above
          6During reaction of two chemical substances , the color produced asses by

          o Spectrophotometer
          o Flame photometer
          7.glycolysis done in :

          o Nucleus
          o Endoplasmic reticulum
          o Mitochondrion
          o Cytoplasm
          o Non of the above
          8.one of the above not present in Diabetic mellitus coma :

          o Hyperglycemia
          o Hypercholesterolemia
          o High number of ketene in urine
          o Non of the above
          9.in which substance give blood glucose when hydrolysis :

          o Muscle glycogen
          o Liver glycogen
          o Heart glycogen
          o Unsaturated fat
          o Triglyceride
          10.in Anti human globulin test do wash of RBC because all serum contain :

          o Albumin
          o a_glubulin
          o b_ globulin
          o fibrinogen
          o Immunoglobulin
          11. the most source of ATP in cell is :

          o Mitochondria
          o Cytoplasm
          o Nucleus
          o Cell wall
          12 . H Ag present in :

          o Pilli
          o Capsule
          o Flagella
          o Cell membrane
          13. urea is final product of catabolism of:

          o amino acid
          o triglyceride
          o cholesterol
          o polysaccharide
          14.Which of White blood cell give immunoglobulin :

          o lymphocyte
          o Neutrophil
          o Basophile
          o Monocyte
          o Eosinophil
          15.Blood of Microfilaria infection diagnosis with :

          o Wuchereia bancofti
          16.one of the immunoglobulin present in trace amount in serum :

          o IgG
          o IgE
          o IgM
          o IgD
          o IgA
          17.gram _ve bacteria colonies color appear :

          o Dark purple
          o Pale to dark red
          o Orange
          18. alkali burn treatment by neutralization with :

          o Sodium bicarbonate powder
          o Boric acid
          o Acetic Acid
          o Cold water
          o Hot water
          19.reason of false positive ABO in compatibility:

          o Cold agglutination
          يتبع

          III-هذا من شخص آخر يسامحني لا أذكر اسمه:

          1- Vitamin K antagonist :
          a- warfarin
          b- Heparin
          c- Protein C
          d- Antithrombin III
          2- One of the intrinsic pathway
          a- factor XI
          b- factor XIII
          c- factor I
          d- factor VII
          3- Para hemophilia is the deficiency of
          a- factor VIII
          b- factor IX
          c- factor V
          d- factor VII
          4- Eosinophilia is seen in :
          a- food sensitivity
          b- Drug sensitivity
          c- Atopic dermatitis
          d- all of the above
          5-Multiple myeloma is a neoplastic proliferation of:
          a- lymphocytes
          b- Granulocytes
          c- Plasma cells
          d-Monocytes
          6- Test for intrinsic pathway:
          a- bleeding time
          b- Thrombin time
          c- Prothrombin time
          d- Partial thromboplastin time
          7- Paul-Bunnel test is done to diagnose:
          a- multiple myeloma
          b- Hodgkin’s disease
          c- Infectious mononucleosis
          PDF created with pdfFactory Pro trial version www.pdffactory.com
          d- all of the above
          8- increased platelet count is :
          a- thrombocytopenia
          b- thrombopoietin
          c- thrombocytosis
          d- all of the above
          9- Decreased platelet count is:
          a- thrombocytopenia
          b- Thrombopoietin
          c- Thrombocytosis
          d- all of the above
          10- All these are causes of thrombocytopenia except:
          a- cytotoxic drugs
          b- Aplastic anemia
          c- Hemorrhage
          d- Radiotherapy
          11- Prothrombin time is done to test:
          a- Intrinsic pathway only
          b- Extrinsic pathway only
          c- Extrinsic and common pathways
          d- Intrinsic and common pathways
          12- Normal bleeding time by Duke’s method:
          a- 2-7 minutes
          b- 2-7 seconds
          c- 2-4 minutes
          d- 2-4 seconds
          13- Normal partial thromboplastin time (PPT) is :
          a- 3-4 minutes
          b- 30-45 seconds
          c- 12-15 seconds
          d- 12-15 minutes
          PDF created with pdfFactory Pro trial version www.pdffactory.com
          14- Hemophilia A is the deficiency of :
          a- factor V
          b- factor VIII
          c- factor IX
          d- all of the above
          15-the most common form of leukemia in children is:
          a- acute lymphoblastic leukemia
          b- Chronic lymphocytic leukemia
          c- Acute myeloid leukemia
          d- Chronic myeloid leukemia
          16- Bence-Jones protein is present in cases of:
          a- chronic myeloid myeloma
          b-acute myeloid myeloma
          c- Hodgkin’s lymphoma
          d- multiple myeloma
          17- Reed-Sternberg cells are found in cases of :
          a- acute lymphoblastic leukemia
          b- Non Hodgkin’s lymphoma
          c- Hodgkin’s lymphoma
          d- Multiple myeloma
          18- Normal platelet count is :
          a- 150,000 to 450,000/min3
          b- 400,000 to 800,000/min3
          c- 4,000 to 11,000 /min3
          d- 50,000 to 100,000/min3
          19- Antithrombin III inhibits:
          a- factor Va
          b- factor VIIIa
          c- factor Xa
          d- all of the above
          20- Heparin potentiate the action of :
          a- protein C
          b- protein S
          c- antithrombin III
          PDF created with pdfFactory Pro trial version www.pdffactory.com
          d- warfarin
          21- Factor II of blood clotting is:
          a- Christmas factor
          b- Fibrinogen
          c- Prothrombin
          d- Thromboplastin
          22- One of fibrinogen group is :
          a- II
          b-V
          c- VII
          d- IX
          23- Fibrinogen is converted to soluble fibrin by:
          a- prothrombin
          b- Thromboplastin
          c- Thrombin
          d- all of the above
          24- Thrombopoitin control the formation of:
          a- red blood cells
          b-White blood cells
          c- platelets
          d- non of the above
          25- Normal prothrombin time (PT) is:
          a- 30-45 seconds
          b- 30-45 minutes
          c- 12-15 seconds
          d- 12-15 minutes
          26- Parasitic disease is associated with:
          a- monocytosis
          b- Lymphocytosis
          c- Basophilia
          d- Eosinophilia
          PDF created with pdfFactory Pro trial version www.pdffactory.com
          27- Philadelphia chromosome is diagnostic for:
          a- acute lymphoblastic leukemia
          b- Acute myeloid leukemia
          c- Chronic lymphocytic leukemia
          d- chronic myeloid leukemia
          28- Normal fibrinogen level:
          a- 150-400 gm%
          b- 150-400 mg%
          c- 15-40 mg%
          d- 15-40 gm%
          29-infectious mononucleosis is caused by:
          a- echo virus
          b- coxsaki virus
          c- Epstein Barr virus
          d- Cytomegalo virus
          30- Atypical lymphocytosis is seen in cases of:
          a- Hodgkin’s lymphoma
          b-Multiple myeloma
          c- Infectious mononucleosis
          d- Chronic lymphocytic leukemia
          31-monospot test is done to diagnose:
          a- Acute myeloid leukemia
          b- Acute lymphoblastic leukemia
          c- Infectious mononucleosis
          d- Infectious lymphocytosis
          32- The test which depend on blood platelets & capillary fragility is:
          a- prothrombin time
          b- Thrombin time
          c- Bleeding time
          d- Clotting time
          33- Fibrin is broken to fibrin degradation products by the action of:
          a- Prothrombin
          b- Thrombin
          c- Plasminogen
          PDF created with pdfFactory Pro trial version www.pdffactory.com
          d- Plasmin
          34- Acute myeloid leukemia is characterized by:
          a- low neutrophil alkaline phosphatase
          b-Myeloblast with Auer rods
          c- Neutrophil with Pleger-Huet anomaly
          d- all of the above
          35- Plasminogen is converted to plasmin by :
          a- Heparin
          b-Histamine
          c- Urokinase
          d- Serotonin
          36- Increase D-dimers and fibrin degradation products are seen in cases of:
          a- Hemophilia A
          b- Vitamin K deficiency
          c- Diffuse intravascular coagulation
          d- Von Willebrand disease
          37- Activated protein C degrades:
          a- factor IXa
          b- Factor VIIIa
          c- Factor Xa
          d- Factor Xia
          38- Heparin is found in
          a- Neutrophil
          b- Basophil
          c- Acidophil
          d- all of the above
          39- In hemophilia A the test which will be prolonged is
          a- PT
          b- PTT
          c- Bleeding time
          d- all of the above
          PDF created with pdfFactory Pro trial version www.pdffactory.com
          40- Bleeding due to overdose of heparin is managed by giving:
          a- Vit K
          b- Vit C
          c- Vit A
          d- Protamin sulphate
          41- Streptokinase and staphylokinase convert:
          a- Prothrombin to thrombin
          b- Fibrinogen to fibrin
          c- Soluble fibrin to insoluble fibrin
          d- Plasminogen to plasmin
          42- Test for platelet function:
          a- Clot retraction
          b- Platelet aggregation
          c- Platelet adhesion
          d- all of the above
          43- Prolonged PT occurs in cases of deficiency of:
          a- Factor III
          b- Factor IV
          c- FactorV
          d- all of the above
          44- normal thrombin time (TT):
          a- 30-45 sec
          b- 2-4 min
          c- 3-9 min
          d- 10-20 sec
          45- cause of vitamin K deficiency:
          a- Obstructive jaundice
          b- Prolonged use of antibiotics
          c- Inadequate intake
          d- all of the above
          46- Cause of Hyper- Coagulable state:
          a- Aplastic anemia
          b- Cytotxic drugs
          c- Polcythemia
          PDF created with pdfFactory Pro trial version www.pdffactory.com
          d- Radiotherapy
          47-Physiological cause of neutrophilia:
          a- New born
          b- Radiotherapy
          c- Cytotoxic drugs
          d- Prolonged use of antibiotics
          48- Leucocytosis characterized by the presence of immature cells and high
          neutrophil alkaline phosphatase:
          a- chronic myeloid leukemia
          b- Acute myeloid leukemia
          c- Leukaemid reaction
          d- non of the above
          49- Normal coagulation time (CT):
          a- 3-9 min
          b- 3-9 sec
          c- 30-40 sec
          d- 30-40 min
          50- The test which measures the clotting time of citrated plasma accelerated
          by the addition of a clotting factor activator (kaolin) , phospholipids and
          calcium:
          a- coagulation time
          b- Prothrombin time
          c- Partial thromboplastin time
          d- Thrombin time
          51- The test which measures the clotting time of citrated plasma to which
          thromboplastin and calcium has been added:
          a- thrombin time
          b- Prothrombin time
          c- Coagulation time
          d- Partial thromboplastin time
          52- The test which is widely used as a control and follow up test to control
          anticoagulant treatment:
          a- APTT
          b- PTT
          PDF created with pdfFactory Pro trial version www.pdffactory.com
          c- PT
          d- TT
          53- A disease characterized by progressive neoplastic proliferation of
          immature white cell precursor:
          a- acute leukemia
          b- Chronic leukemia
          c- Lymphoma
          d-Multiple myeloma
          54- The absolute lymphocyte count may be up to 300,000 or more between 70
          and 90%of white cells in the blood film appear as small lymphocytes . THE
          CASE IS:
          a- Acute myeloid leukemia
          b- Acute lymphoblastic leukemia
          c- Chronic myeloid leukemia
          d- Chronic lymphocytic leukemia
          55- Variation in red cells size:
          a- Poikilcytosis
          b- Anisocytosis
          c- Reticulocytosis
          d- Leukocytosis
          56- Dark red cells with no area of central pallor:
          a- Stomatocyte
          b- Sherocyte
          c- Acathocyte
          d- Schistocyte
          57-Microcytic hypochromic anemia
          a- hereditary spherocytosis
          b- Sickle cell anemia
          c- Iron deficiency anemia
          d- Vit B12 deficiency anemia
          58- Target cells are seen in cases of:
          a- folic acid deficiency
          b- Iron deficiency anemia
          c- Vit B12 deficiency anemia
          PDF created with pdfFactory Pro trial version www.pdffactory.com
          d- Thalassemia
          59- Red cells with elongated area of central pallor:
          a- spherocyte
          b- Schistocyte
          c- Stomatocyte
          d- Elliptocutes
          60- ……………….. Symmetric, short , sharp projection from the red cells and
          seen in iron deficiency anemia:
          a- echinocyte
          b- Acanthocyte
          c- Elliptocyte
          d- Ovalocyte
          61- ………………is a condition in which the absorption of vit B12 is greatly
          impaired due to failure or marked reduction of intrinsic factor secretion:
          a- fauvism
          b- fanconi’s anemia
          c-sickle cell anemia
          d- thalassemia
          62- Hyperchromic cells are seen in:
          a- iron deficiency anemia
          b- Thalassemia
          c- Hereditary spherocytosis
          d- Sickle cell anemia
          63- A prolonged low rate of bleeding results in:
          a- normochromic anemia
          b- Hypochromic anemia
          c- Hyperchromic anemia
          d- non of the above
          64- Schilling test is done in diagnosis of:
          a- iron deficiency anemia
          b- Pernicious anemia
          c- Aplastic anemia
          d- folic acid deficiency
          PDF created with pdfFactory Pro trial version www.pdffactory.com
          65- Defective synthesis of either alpha or beta chains of normal hemoglobin
          cause:
          a- sickle cell anemia
          b- Aplastic anemia
          c- Pernicious anemia
          d- Thalassemia
          66- Neutrophils represent……………of circulating leukocyte:
          a- 2-8%
          b- 0-1%
          c- 50-70 %
          d- 2-4%
          67- …………. are non nucleated, biconcave shaped cells:
          a- platelet
          b- Leukocyte
          c- Erythrocyte
          d- Macrophages
          68- …………….. represent 50-70%of total leukocytes
          a- lymphocytes
          b- Neutrophils
          c- Monocytes
          d- Eosinophilis
          69- ……………have a characteristic biffed nucleus and their cytoplasm is
          filled with large refractile granules that stain red in blood smear
          a- neutrophils
          b- Eosinophilis
          c- Basophiles
          d- Lymphocytes
          70- The cell which is responsible for antibody production is:
          a- moncytes
          b- T-lymphocytes
          c- B-lymphocytes
          d- Neutrophils
          PDF created with pdfFactory Pro trial version www.pdffactory.com
          71- ……is a curved cell with sharp ends seen in haemoglobinopathies (HBS)
          a- sickle cell
          b- Spherocyte
          c- Ovalocyte
          d- Stomatocyte
          72- All of the following is correct about sickle cell anemia except:
          a- leg ulcers
          b- Gall stones
          c- Enlargement of spleen
          d- Attacks of pain
          73- ……….. is a single, large, rounded , dark , purple remnant of nucleus
          a- Heinz body
          b- Howeel-Jolly body
          c- Pappenheimer body
          d- Cabot ring
          74- Agranulocyte:
          a- neutrophil
          b- Lymphocyte
          c- Basophile
          d- Eosinophil
          75- Pica ( craving to eat unusual substance such as clay or ice) is one of the
          symptoms of:
          a- G6PD deficiency
          b- Thalassemia
          c- Megaloblastic anemia
          d- Iron deficiency anemia
          76- In…………….. there’s a decreased or absent hemosiderin in bone
          marrow
          a- sideroblastic anemia
          b- Iron deficiency anemia
          c- Megaloblastic anemia
          d- Hemolytic anemia
          PDF created with pdfFactory Pro trial version www.pdffactory.com
          77- Chloramphenicol may cause …………. anemia in long term therapy
          a- iron deficiency
          b- Vit B12 deficiency
          c- folic acid deficiency
          d- Aplastic anemia
          78- ………. is the fluid (with anticoagulant) component of blood , it contains
          salt & organic compounds:
          a- plasma
          b- Serum
          c- Hemoglobin
          d- Billirubin
          79- Poikilocytosis is:
          a- variation in red cell size
          b- Variation in red cell color
          c- Variation in red cell shape
          d- non of the above
          80- Red cell fragments:
          a- echinocyte
          b- Elliptocyte
          c- Schistocyte
          d- Stomatocyte
          81- It is a defect of red cell member
          a- Thalassemia
          b- Sickle cell anemia
          c- Hereditary spherocytosis
          d- Megaloblastic anemia
          82- All of the following is correct regarding spherocytosis except:
          a- normocytic normochromic anemia
          b- Decreased reticulocyte count
          c- Raised plasma bilirubin
          d- Increased osmotic fragility
          PDF created with pdfFactory Pro trial version www.pdffactory.com
          83- Heinz bodies are seen in cases of
          a- hereditary spherocytosis
          b- Hereditary elliplocytosis
          c- G6PD deficiency
          d- sickle cell anemia
          84- ………………is caused by substitution of amino acid (valine) instead of
          glutamic acid at position No.#6 in the beta chain of hemoglobin
          a- Hb-A
          b- Hb-A2
          c- Hb –F
          d- Hb –S
          85- Atrophy of the spleen is seen in cases of:
          a- Thalassemia
          b- Sickle cell anemia
          c- G6PD deficiency
          d- Hereditary elliplocytosis
          86- iron deficiency lead to :
          a- normocytic normochromic anemia
          b- microcytic hypochromic anemia
          c- macrocytic anemia
          d- hemolytic anemia
          87- Neurological symptoms are seen in cases of:
          a- iron deficiency anemia
          b- folic acid deficiency
          c- Vit B12 deficiency
          d- all of the above
          88-Megaloplastic hematopoiesis is seen in cases of:
          a- iron deficiency anemia
          b- folic acid deficiency
          c- Vit B12 deficiency
          d- Vit C deficiency
          PDF created with pdfFactory Pro trial version www.pdffactory.com
          89- Fanconi’s anemia is a type of :
          a - vit B12 deficiency
          b- aplastic anemia
          c- Thalassemia
          d- folic acid deficiency anemia
          90- the most abundant leukocyte in a normal blood smear of adult is :
          a- lymphocyte
          b-Monocyte
          c- Eosinophil
          d- Neutrophil
          91- The first line of defense against parasites:
          a- neutrophils
          b- Basophile
          c- Eosinophil
          d- Lymphocyte
          92- …………….. play a role in immediate and delayed hypersensitivity:
          a- monocyte
          b- Lymphocyte
          c- Eosinophil
          d- Basophile
          93- The largest leukocyte is :
          a- neutrophils
          b- Lymphocyte
          c-Monocyte
          d- Basophile
          94- Cell which participate in cell mediated immunity:
          a- monocyte
          b- B- lymphocyte
          c- T- lymphocyte
          d- neutrophils
          PDF created with pdfFactory Pro trial version www.pdffactory.com
          95- ………promotes blood clotting and help to prevent blood loss from
          damaged blood vessels:
          a- platelets
          b-WBCs
          c- RBCs
          d- all of the above
          96-antibody induced hemolytic disease in new born that is caused by blood
          group incompatibility between mother and fetus:
          a- hemolytic uremic syndrome
          b- Erythroblastosis fetalis
          c- Hereditary spherocytosis
          d- Thromboloc thrombocytopenic purpurea
          97- Young red blood cell with cytoplasmic RNA:
          a- spherocyte
          b- Reticulocyte
          c- Stomatocyte
          d- elliptocyte
          98- Normal adult hemoglobin tetramer is:
          a- 2 alpha : 2 gama
          b- 2 alpha : 2 beta
          c- 2 alpha : 2 delta
          d- 2 beta : 2 gama
          99- ……………represent 2-4 %of total leukocyte:
          a- neutrophils
          b- Basophile
          c- Eosinophil
          d-Monocyte
          100- …………are small cytoplasmic fragment derived from megakaryocytic:
          a- RBCs
          b-WBCs
          c- Platelet
          d- non of the above
          PDF created with pdfFactory Pro trial version www.pdffactory.com
          101 -………….. is the reduction in the amount of circulating hemoglobin ,
          red blood cells or both:
          a- polycythemia
          b- Anemia
          c- Hemophilia
          d- Leucopenia
          102- Thalassemia is :
          a- microcytic anemia
          b- Macrocytic anemia
          c- Normocytic anemia
          d- non of the above
          103- Vit B12 deficiency lead to :
          a- hemolytic anemia
          b- Microcytic anemia
          c- Normocytic anemia
          d- Megaloblastic anemia
          104- Lymphocyte represent ………….. of total leukocyte:
          a- 20-40%
          b- 50-70%
          c- 2-8%
          d- 1-5 %
          105- Plumer- Vinson syndrome may be seen in cases of:
          a- iron deficiency anemia
          b- Vit B12 deficiency anemia
          c- Aplastic anemia
          d- folic acid deficiency anemia
          106- peripheral, pale inclusions that push out the cell membrane and
          composed of hemoglobin:
          a- cabot ring
          b- Pappenheimer body
          c- Howell-Jolly body
          d- Heinz body
          PDF created with pdfFactory Pro trial version www.pdffactory.com
          107- it’s an acute hemolytic anemia occurring after the ingestion of broad
          bean in individual with deficiency of G6PD :
          a-thalassemia
          b- Favism
          c- Fanconi’s anemia
          d- Cooley’s anemia
          108-………………. is a multiple small , peripheral grape like purple clusters
          of iron:
          a- cabot ring
          b- Heinz body
          c- Howell-Jolly body
          d- Pappenheimer body
          109- the blood smear gives the physician information concerning:
          a- morphology of RBCs and platelet
          b- Presence of abnormal inclusion
          c- Presence of immature cells
          d- all of the above
          110- hypersplenism is one of the causes of :
          a- iron deficiency anemia
          b- Hemolytic anemia
          c- Aplastic anemia
          d- Megaloblastic anemia
          111- Increased reticulocytes count is seen in cases of:
          a- hereditary spherocytosis
          b- G6PD deficiency
          c- Sickle cell anemia
          d- all of the above
          112- The antibody which can pass the placenta:
          a- IgM
          b- Ig G
          c- IgD
          d- Ig E
          PDF created with pdfFactory Pro trial version www.pdffactory.com
          113- ………is an autoimmune disease in which there is an immune
          destruction of the acid and pepsin secreating cells of the stomach:
          a- fanconi’s anemia
          b- cooley’s anemia
          c-pernicious anemia
          d- non of the above
          114- All of these are laboratory features of aplastic anemia except:
          a- pancytopenia
          b-markedly hypocellular marrow
          c- Increased reticulocyte count
          d-Markedly increase in serum erythropoietin
          115- Secondary granules of neutrophils contain:
          a- elastase
          b- Myeloperoxidase
          c- Lysozyme
          d- Histamine
          116- Monocytes represent ………………. of total leukocyte:
          a- 0-1%
          b- 2-4%
          c- 2-8%
          d- 20-4 %
          117- the reagent used for leukocyte count is :
          a- citric acid
          b- Acetic acid
          c- Hydrochloric acid
          d- Sulphoric acid
          118- All are Prokaryotic cells except:
          a- Fungi
          b- Bacteria
          c - Chlamydia
          d- Mycoplasma
          PDF created with pdfFactory Pro trial version www.pdffactory.com
          119- Viruses:
          a- Contain only DNA or RNA
          b - They Contain ribosome
          c- Did not affected by antibiotics
          d- a+c
          120- All of these are essential structure except:
          a- Nuclear body
          b- Spores
          c- Cell wall
          d- Plasma Membrane
          121- ______________ is giving the shape to the bacteria
          a.- Cytoplasmic Membrane
          b- Capsule
          c- CellWall
          d- All of the above
          122- One of its functions is selective permeability
          a- Cell wall
          b- Plasma membrane
          c- Capsule
          d- Spores
          123- They are responsible for Haemagglutination Phenomenon
          a- Flagella
          b- Fimbria
          c- Capsule
          d- Cell wall
          124- Clostridium Tetani is:
          a- Atrichous bacteria
          b-Mono-trichous bacteria
          c- Amphi-trichous bacteria
          d- Peri-trichous bacteria
          PDF created with pdfFactory Pro trial version www.pdffactory.com
          125- Short curved or straight rods, motile by single polar flagellum
          a- spirochaeta
          b- Vibrio
          c- Escherichia
          d- Lactobacillus
          126- Small gram negative cocci, occur in pairs
          a- staphylococcus
          b- streptococcus
          c- neisseria
          d- non of the above
          127- Transfer of genetic information from one bacterium to another by
          bacteriophages is:
          a. Transformation
          b. Tansduction
          c. Conjugation
          d. Mutation
          128- Salmonella are:
          a- Obligatory Aerobic bacteria
          b.- Obligatory Anaerobic bacteria
          c- Facultative Anaerobic bacteria
          d- Micro-aerophilic bacteria
          129- According to pH, vibrio cholera is
          a- Osmophilic bacteria
          b- Basophilic bacteria
          c- Acidophilic bacteria
          d- Neutrophilic bacteria
          130- Staphylococci are:
          a- Atrichous bacteria
          b- Mono-trichous bacteria
          c- Amphi-trichous bacteria
          d- Peri-trichous bacteria
          PDF created with pdfFactory Pro trial version www.pdffactory.com
          131- During replication of DNA, copying errors may occur and this is called
          a- Conjugation
          b- Transduction
          c- Transformation
          d- Mutation
          132- Obligatory Anaerobic bacteria
          a- grow only in presence of oxygen
          b- grow only in absence of oxygen
          c- grow either in presence or absence of oxygen
          d- grow in presence of oxygen traces and 5 – 10% CO2
          133- Neutrophilic bacteria grow well at
          a- pH 8.5 – 9.0
          b- pH 7.2 – 7.4
          c- pH 5.0 – 5.5
          d- pH 2.5 – 3.0
          134- The rate of cell death increase and bacterial growth stopped, this is
          a- Adaptation phase
          b- Exponential phase .
          c- Stationary phase.
          d- Decline Phase
          135- Beta-hemolytic
          a- Cause complete hemolysis of RBC’s
          b- Cause chemical change of Hemoglobin in RBC’s
          c- Do not cause hemolysis
          d- None of them
          146- Mesophilic bacteria grow at:
          a- 37°C
          b- 14°C
          c- 60°C
          d- 120°C
          PDF created with pdfFactory Pro trial version www.pdffactory.com
          137- Circulation of Bacteria and its toxins in blood
          a- Pyaemia
          b- Toxemia
          c.- Bacteremia
          d- Septicemia
          138- Disinfections that applied on living or injured tissues:
          a- Sterilization
          b- Antiseptic
          c- Sanitation
          d- Decontamination
          139- Hot air oven is used for sterilization of
          a- Glass
          b- Rubber Gloves
          c.- Plastic Syringes
          d- Catheters
          140- Disinfectant for superficial fungal infection
          a- Phenol
          b- Potassium permanganate
          c- Chlorine
          d- Hypochlorite compounds
          141- Rifampin
          a- inhibit cell wall synthesis
          b- inhibit protein synthesis
          c- inhibit folic acid pathway
          d- inhibit mRNA synthesis
          142- Transacetylase inactivate
          a- aminoglycosides
          b- chloramphenicol
          c- penicillin
          d- cephalosporins
          PDF created with pdfFactory Pro trial version www.pdffactory.com
          143- The color of gram positive bacteria is
          a- Yellow
          b- Black.
          c- Pink
          d- Violet
          144- Selective media for fungi
          a- blood agar
          b- Mac Conkey agar
          c- Nutrient agar
          d- Sabourand’s dextrose agar
          145- Histoplasma is a :
          a- Systemic mycosis
          b- Sub – Cutaneous mycosis
          c- Cutaneous mycosis
          d- Superficial mycosis
          146- All are asexual spores produced by mould except
          a- Conidio – spores
          b- Sporangio – spores
          c- Endospores
          d- Arthro – spores
          147- They reproduce only by Asexual reproduction
          a- Blastomycosis
          b- Deutromycosis
          c- Ascomycetes
          d- Zygomycetes
          148- The functions of cell wall is all of the following except:
          a- Giving the shape to the bacteria
          b- Carrying somatic antigen
          c- Selective permeability and transport of solutes
          d- Protect the bacteria from plasmolysis
          PDF created with pdfFactory Pro trial version www.pdffactory.com
          149- Atrichous Bacteria are:
          a- Bacteria contain one flagellum
          b- Bacteria contain 2 flagella
          c- Bacteria without flagella
          d- Bacteria with a tuft of flagella
          150- ________________ are essential for host cell attachment:
          a- Flagella
          b- Fimbria
          c- Spores
          d- Capsules
          151- Irregular clusters of spherical cells:
          a- Streptococcus
          b- Staphylococcus
          c- Lactobacillus
          d- Escherichia
          152- Clostridium Botulinum is:
          a- Obligatory Aerobic Bacteria
          b- Facultative Anaerobic Bacteria
          c- Obligatory Anaerobic Bacteria
          d- Micro- aerophilic Bacteria
          153- Basophilic Bacteria grow well at:
          a- pH 8.5 – 9.0
          b- pH 7.2 – 7.4
          c- pH 5.0 – 5.5
          d- None of the above
          154- Cells are divided at high & constant rate:
          a- Decline Phase
          b- Stationary Phase
          c- Log Exponential Phase
          d- Adaptation Phase
          155- The dominant bacterial species in dental plaque are:
          a- Coagulase Negative Staphylococci
          b- Lactobacillus
          c- Bacteroides
          d- Streptococcus Sanguis
          PDF created with pdfFactory Pro trial version www.pdffactory.com
          156- The spread of Pyogenic Bacteria in blood stream to different organs &
          produce multiple abscess is:
          a- Septicemia
          b- Bacteremia
          c-Toxemia
          d- Pyaemia
          157- Inhibit the growth of micro organisms
          a- Bacteriostatic
          b- Bactericidal
          c- Fungicidal
          d- Germicidal
          158- To sterilize fluid damaged by heat:
          a- Gaseous Sterilization
          b- Heat Sterilization
          c- Filtration
          d- Ionizing Radiation
          159- For water disinfection we use:
          a- Hydrogen peroxide
          b- Formaldehyde
          c- Chlorine
          d- Hypochlorite compounds
          160- Mechanism of action of penicillin:
          a- Block peptidoglycan synthesis
          b- Inhibit peptidyglycan cross – linking
          c- inhibit folic acid pathway
          d- inhibit protein synthesis
          161- Sulfonamides:
          a- inhibit cell wall synthesis
          b- inhibit protein synthesis
          c- inhibit DNA synthesis
          d- inhibit folic acid pathway
          PDF created with pdfFactory Pro trial version www.pdffactory.com
          162- Acetylase inactivates:
          a- B – Lactam antibiotics
          b- Aminoglycosides
          c- Cloramphenicol
          d- All of the above
          163- Ringworm disease is caused by
          a- Zygomycetes
          b- Ascomycetes
          c- Blastomycosis
          d- None of the above
          164- For wet – mount technique we add:
          a- NaoH
          b- K oH
          c- H2 O2
          d- All of the above
          165- They are transmitted by arthropods
          a- Chlamydia
          b- Spirochetes
          c- Mycoplasma
          d- All of the above
          166- In the past they were listed as large viruses
          a- Richettsia
          b- Mycoplasma
          c- Chlamydia
          d- None of the above
          167- Bacteria multiply by:
          a- Replication cycle
          b- Simple binary fission
          c- Sexual reproduction
          d- All of the above
          168- It protects bacteria from antibiotics
          a- Capsule
          b- Cell membrane
          c- Flagella
          d- Fimbria
          PDF created with pdfFactory Pro trial version www.pdffactory.com
          169- Vibro cholera is:
          a-Mono –trichous bacteria
          b- Atrichous bacteria
          c- Lopho-trichous bacteria
          d- Peri-trichous bacteria
          170- Short rods, motile by peritrichous flagella
          a- Spirochaeta
          b- Lactobacillus
          c- Escherichia coli
          d- Vibrio
          171- To take up soluble DNA fragments derived from other, closely related
          species is:
          a. Mutation
          b. Transformation
          c. Transduction
          d. Conjugation
          172- Tuberculosis are
          a- micro-airophilic
          b- Facultative anaerobic
          c- Obligatory anaerobic
          d- Obligatory aerobic
          173- According to pH, Lactobacillus is
          a- Neutrophlic bacteria
          b- Acidophilic bacteria
          c- Basophilic bacteria
          d- None of the above
          174- Bacteria without cellWall
          a- Chlamydia
          b- Rickettsia
          c- Mycoplasma
          d.- Spirochetes
          PDF created with pdfFactory Pro trial version www.pdffactory.com
          175- Brucella Melitensis is
          a- Obligatory aerobic bacteria
          b- Obligatory anaerobic bacteria
          c- Facultative anaerobic bacteria
          d- Micro-aerophilic bacteria
          176- Pseudomonas aeroginosa is
          a- Peri-trichous bacteria
          b- Lopho-trichous bacteria
          c- Amphi-trichous bacteria
          d- Monotrichous bacteria
          177- Genetic information of bacteria is carried on
          a- Messenger RNA
          b- Transfer RN|A
          c- Transcript RNA
          d- Double – Stranded DNA
          178- Thermophilic bacteria grow at
          a- 60 – 80 °C
          b- 0 - 20°C
          c-. 37°C
          d- 100 - 120°C
          179- Acidophilic bacteria grow at
          a- pH 7.2 – 7.4
          b- pH 5.0 – 5.5
          c- pH 8.5 – 9.0
          d- None of the above
          180- Mycoplasma is
          a- Neutrophilic bacteria
          b- Acidophilic bacteria
          c- Basophilic bacteria
          d- All of the above
          PDF created with pdfFactory Pro trial version www.pdffactory.com
          181- It is the adaptation of bacteria to the fresh medium
          a- Lag phase
          b- Decline phase
          c- Logarithmic Phase
          d- Stationary phase
          182- Bacteria which do not cause hemolysis is
          a- Beta-Hemolytic
          b- Alpha-Hemolytic
          c- Gama Hemolytic
          d- None of the above
          183- Normal flora of Lower intestine are all of the following except:
          a- Staphylococci
          b- Diphtheroids
          c- Shigella
          d- Lactobacillus
          184- Opportunistic pathogens are all of the following except:
          a- Cause a disease when the host defense are suppressed.
          b- Are normal flora of healthy body
          c- Are greatly harmful
          d- Do not invade the body or tissue.
          185- For disinfection of mattresses :
          a- Hot air oven
          b- Autoclave
          c- Ethylene Oxide
          d- Hydrogen Peroxide
          186- Pyschrophilic bacteria grow at:
          a- 10٠°C
          b- 6٠°C
          c- 14°C
          d- 37°C
          PDF created with pdfFactory Pro trial version www.pdffactory.com
          187- Bacteria which contain chlorophyll
          a- Heterotrophic bacteria
          b- Autotrophic bacteria
          c- Photosynthetic bacteria
          d- All of the above
          188- Tricophyton is one of
          a- Yeast
          b- Moulds
          c- Dermatophyte
          d- Dimorphic Fungi
          189- Plastomyces is one of
          a- Dermatophytes
          b- Dimorphic Fungi
          c- Yeast
          d- Moulds
          190- Color of gram negative bacteria is
          a- Violet
          b- Green
          c- Red
          d- Black
          191- Acid Fast Bacteria
          a- Salmonella
          b- Shigella
          c- M. Tuberculosis
          d- E – Coli
          192- Spherical or avoid cells occurring in chains
          a- Staphylococci
          b- Streptococci
          c- Lactobacillus
          d- Spiro chaeta
          PDF created with pdfFactory Pro trial version www.pdffactory.com
          193- ………………carries the genetic information
          a- the envelope
          b- the capsid
          c- the nucleic acid
          d- the prion
          194- ……………………may be seen under light microscope
          a- rota virus
          b- influenza virus
          c- herps virus
          d- pox virus
          195- viruses may be:
          a- monomorphic
          b- pleomorphic
          c- dimorphic
          d- all of the above
          196- viral capside is formed of:
          a- protein
          b- glycogen
          c- lipoprotein bilayer
          d- glycoprotein
          197- class III in Baltimor classification is:
          a- double stranded DNA viruses
          b- single stranded DNA viruses
          c- double stranded RNA viruses
          d- single stranded RNA viruses
          198- Hierarchial virus classification system use the following characters
          except:
          a- nature of nucleic acid
          b- capside symmetry
          c- diameter of viron & capside
          d- virus molecular weight
          PDF created with pdfFactory Pro trial version www.pdffactory.com
          199- in viral replication which is true:
          a- penetration is the 1st step
          b- assembly is the last step
          c- relaease is the last step
          d- all of the above
          200- viron:
          a- may be extracellular phase of virus
          b- may be intracellular phase of virus
          c- can grow and replicate
          d- means “ virus – like “
          201- pleomorphic viruses means :
          a- virus which have constant shape
          b- virus that may appear in 2 forms
          c- virus that have not a constant morphology
          d- virus that have spherical shape
          202- vapor of gold is used in :
          a- shadow casting technique
          b- negative staining technique
          c- positive staining technique
          d- non of the above
          203- direct diagnosis of virus :
          a- ELISA
          b- CFT
          c- IFT
          d- PCR
          204- all of the following are required in cell culture except:
          a- neutral PH
          b- presence of buffer salts
          c- presence of antibiotics
          d- incubation at 20 C
          PDF created with pdfFactory Pro trial version www.pdffactory.com
          205- all of the following are diagnostic molecular biological technique
          except:
          a- PCR
          b- ELISA
          c- nucleic acid hyperdization
          d- DNA finger printing
          206- PCR require all of the following except:
          a- extracted DNA template
          b- 2 specific primers
          c- reation buffer
          d- RNA polymerase
          207- bacteriophage is :
          a- virus that can be killed by antibiotic
          b- virus that act like a bacteria
          c- bacteria that act like a virus
          d- virus that infect bacteria
          208- all of the following viruses are transmitted by blood except:
          a- HIV
          b- HBV
          c- HCV
          d- herps virus
          209-all of the following are RNA viruses except:
          a- corona viridase
          b- reoviridase
          c- picorona viridase
          d- pox viridase
          210- penetration of naked virus is by :
          a- fusion
          b- endocytosis
          c- translocation
          d- all of the above
          PDF created with pdfFactory Pro trial version www.pdffactory.com
          211- amniotic cavity inoculation is one type of virus culture in:
          a- tissue wall
          b- lab animals
          c- embryonated egg
          d- non of the above
          212- …………….. is an invitro method amplification of a short sequence of
          target DNA
          a- PCR
          b- hyberdization
          c- finger printing
          d- all of the above
          213- nucleic acid hyberdization means:
          a- probe anneling or binding with it’s complementary
          segment of NA
          b- fragmentation of nucleic acid
          c- amplification of nucleic acid
          d- non of the above
          214- DNA hyberdization is performed by:
          a- primers
          b- DNA labeled probe
          c- restriction endonuclease
          d- non of the above
          215-PCR starts with :
          a- annealing
          b- denaturation of DNA
          c-extension of primers
          d- non of the above
          216- ……………….. is a piece of DNA fragment of a particular gene that can
          bind specially with it’s complementary piece of DNA:
          a- codon
          b- probe
          c- LCR
          d- code
          PDF created with pdfFactory Pro trial version www.pdffactory.com
          217- how many primers are used in PCR :
          a- non
          b- one
          c- two
          d- three
          218- secreted from virus infected cell:
          a- C-reactive protein
          b- complement
          c- interferons
          d- lysozyme
          219- it’s a process of coating the foreign cell by antibodies :
          a- chemotaxis
          b- adherence
          c- opsonization
          d- endocytosis
          220- the only Ig that can cross the placenta:
          a- IgE
          b- IgD
          c- IgM
          d- IgG
          221- the predominant Ig in primary immune response:
          a- IgM
          b- Ig A
          c- Ig G
          d- Ig D
          222- the predominant Ig in body secretion:
          a- IgM
          b- Ig A
          c- Ig G
          d- Ig D
          PDF created with pdfFactory Pro trial version www.pdffactory.com
          223- it’s a cytophilic Ab
          a- IgM
          b- Ig E
          c- Ig G
          d- Ig D
          224- it’s the main host defense against helminthes infestations:
          a- Ig E
          b- Ig A
          c- Ig G
          d- Ig D
          225- Antibodies are secreted from:
          a- B-lymphocyte
          b- T- lymphocyte
          c- monocyte
          d-Macrophage
          226-immunity through placenta is:
          a- natural active immunity
          b- Artificial active immunity
          c- Natural passive immunity
          d- Artificial passive immunity
          227- one of the primary lymphoid organs:
          a - spleen
          b- Tonsils
          c- Thymus gland
          d- Lymph nodes
          228- Its a memory of T cell mediated immunity:
          a-T – helper
          b- T suppressor
          c- T- cytotoxic
          d- T effector
          PDF created with pdfFactory Pro trial version www.pdffactory.com
          229- all are character of primary immune response except:
          a- stimulate native B- cells
          b- short duration
          c- high level of Ab
          d- low affinity of Abs to Ags
          230- they are cytokines that regulate the interaction between lymphocyte and
          other leukocyte:
          a- interferon
          b- Growth factor
          c- Tumor necrosis factor
          d- Interleukins
          231- Malignant tumor in epithelial cells:
          a- leukemia
          b- Carcinoma
          c- Sarcoma
          d- non of the above
          232- Kaposi’s sarcoma is caused by:
          a- hepatitis B virus
          b- Epstein Barr virus
          c- Cytomegalo virus
          d- Human papilloma virus
          233- Reagenic Ab:
          a- IgM
          b- Ig A
          c- Ig G
          d- Ig E
          234- Tuberculin test is :
          a- type I hypersensitivity
          b- type II hypersensitivity
          c- type III hypersensitivity
          d- type VI hypersensitivity
          PDF created with pdfFactory Pro trial version www.pdffactory.com
          235- Formation of auto-Abs ( IgG) to acetylcholine receptors:
          a- multiple sclerosis
          b- Graves’s disease
          c-Myasthenia gravis
          d- Rheumatoid arthritis
          236- Auto –Abs against Ags on the myelin sheath:
          a- Hashimoto’s thyroditis
          b- Graves’s disease
          c- Myasthenia gravis
          d- Multiple sclerosis
          237- Anti nuclear Abs found in case of
          a- SLE
          b- IMN
          c- CML
          d- AML
          238- If the rejection to kidney grafts is happened after 7 days this is:
          A- Hyper acute rejection
          b- Accelerated rejection
          c- Acute rejection
          d- Chronic rejection
          239- The test which determine the presence of Ab to HIV Ags(p24,P31)is:
          a- ELISA test
          b- immunfluoresence test
          c- western blot test
          d- P24 Ag test
          240- Vaccination is:
          a- natural active immunity
          b- Artificial active immunity
          c- Natural passive immunity
          d- Artificial passive immunity
          PDF created with pdfFactory Pro trial version www.pdffactory.com
          241- Lysozyme found in high concentration:
          a- urine
          b- Sweat
          c- Tears
          d- CSF
          242- ………….. inhibit virus replication:
          a- acute phase protein
          b- Lysozyme
          c- Complement
          d- Interferon
          243- it’s a foreign substance of low molecular weight which can not induce
          specific immune response unless it’s conjugated with a carrier:
          a- antigen
          b- Immunogene
          c- Hatpin
          d- Adjuvant
          244-the main Ig in secondary immune response:
          a- Ig A
          b- Ig G
          c- IgD
          d- IgM
          245- Wassermantest is done for the diagnosis of :
          a- T.B
          b- Syphilis
          c- Diphtheria
          d- all of the above
          246- Elek test is done for the diagnosis of:
          a- T.B
          b- Syphilis
          c- Diphtheria
          d- Gonorrhea
          PDF created with pdfFactory Pro trial version www.pdffactory.com
          247- thyrotoxin is secreted from:
          a- pituitary gland
          b- Thyroid gland
          c- Suprarenal gland
          d- non of the above
          248-they are material that enhance the normal immune response non
          specifically:
          a- antigen
          b- Hapten
          c- Adjuvant
          d- Mitogen
          249- they increase the mitotic activity of lymphocyte:
          a- immunogens
          b- Adjuvant
          c- Hapten
          d- Mutagen
          250- it’s the most efficient Ig in complement fixation:
          a- IgM
          b- Ig A
          c- Ig G
          d- Ig E
          251- complement concentration increased in:
          a- nephritis
          b- Serum sickness
          c- Carcinoma
          d-Malaria
          252- the acceptable minimum concentration of heamoglobin for the donor is:
          a- 12-5 g/dl
          b- 10-5 g/dl
          c- 9.5 g/dl
          d- 15.5 g/dl
          PDF created with pdfFactory Pro trial version www.pdffactory.com
          253- all of these are adverse donor reaction except :
          a- syncope
          b- convulsions
          c- hypertension
          d- nausea and vomiting
          254- the bleeding room should be equipped with the following injections
          ……. except:
          a- adrenaline
          b- atropine
          c- dexamethasone
          d- calcium gluconate
          255- the ****f life of deglycerolized blood is:
          a- 24 hours
          b- 48 hours
          c- 72 hours
          d- 12 hours
          256- the platelet are stored at :
          a- 2 : 6 c ْ
          b- 20 : 26 c ْ
          c- 20 : 50 c ْ
          d- 120 : 180 c ْ
          257- major cross match:
          a- donor’s cells + patient serum
          b- donor’s serum + patient’s cell
          c- donor’s cells + donor’s serum
          d- patient’s cells + patient’s serum
          258- cold agglutination syndrome is due to :
          a- IgG
          b- IgM
          c- IgA
          d- Ig D
          PDF created with pdfFactory Pro trial version www.pdffactory.com
          259- the antibody usually responsible for PCH is:
          a- anti A
          b- anti B
          c- anti Po
          d- anti D
          260- the worm autoimmune hemolytic anemia is due to :
          a- IgM
          b- IgA
          c- Ig G
          d- Ig D
          261- HLA antigen is controlled by a region on chromosome No.:
          a- 9
          b- 6
          c- 12
          d- 3
          262- the blood group individuals which are found to be resistance to a west
          African malaria parasite:
          a- kell
          b- kidol
          c- duffy
          d- Lutheran
          263- the color of anti-sera A:
          a- yellow
          b- colorless
          c- blue
          d- green
          264- the color of anti-sera B is:
          a- red
          b- blue
          c- yellow
          d- colorless
          PDF created with pdfFactory Pro trial version www.pdffactory.com
          265- detecting ABO grouping by using the cells of the patient is:
          a- reverse grouping
          b- forward grouping
          c- backward grouping
          d- non of the above
          266- detecting ABO grouping using the serum of the patient is
          a- reverse grouping
          b- forward grouping
          c- backward grouping
          d- non of the above
          267- patient with anti A antibodies in his serum , his blood group is :
          a- group A
          b- group B
          c- group AB
          d- groupO
          268-the red cells of a patient with neither A nor B antigen on the surface , his
          blood group is :
          a- group A
          b- group B
          c- group AB
          d- groupO
          269- cryoprecipitated is used for the treatment of :
          a- hemophilia A
          b- anemia
          c- leukemia
          d- thrombocytopenia
          270- universal donor is :
          a- group A
          b- group B
          c- group AB
          d- groupO
          PDF created with pdfFactory Pro trial version www.pdffactory.com
          271-universal recipient is :
          a- group A
          b- group B
          c- group AB
          d- groupO
          272- fresh frozen plasma is used for the treatment of the following…….
          except:
          a- DIC
          b- vit K deficiency
          c- clotting factor deficiency
          d- protein replacement
          273- the indication for exchange transfusion :
          a- disseminated intravascular coagulation
          b- immune thrombocytopenic purpura
          c- hemolytic disease of newborn
          d- vit K deficiency
          274- a case of anemia with coronary disease must be treated with:
          a- whole blood
          b- packed red cells
          c- fresh frozen plasma
          d- cryoprecipitate
          275- the storage time of acid citrate dextrose (ACD) is:
          a- 42 days
          b- 21 days
          c- 35 days
          d- 15 days
          276- the storage time of saline adenine glucose – mannitol (SAGM) is:
          a- 42 days
          b- 21 days
          c- 35 days
          d- 15 days
          PDF created with pdfFactory Pro trial version www.pdffactory.com
          277- to collecte blood from the donor , you have to apply the blood pressure
          cuff on his arm and raise the pressure to:
          a- 150 – 180 mmHg
          b- 15 -16 mmH
          c- 50 -60 mmHg
          d- non of the above
          278- all of these are transfusion transmitted disease except:
          a- hepatitis B
          b- hepatitis C
          c- hepatitis A
          d- AIDS
          279- the donor is permanently deferred if gives a positive history of :
          a- German measles
          b- antidiphtheria vaccine
          c- epilepsy
          d- tattooing
          280- the blood donor must not be less than:
          a- 30 years
          b- 18 years
          c- 40 years
          d- 25 years
          281- the clinical significance of Rh antigen :
          a- blood transfusion
          b- hemolytic disease of newborn
          c- medico legal
          d- all of the above
          282- in cross matching we use:
          a- albumin
          b- antihuman globulin
          c- papain enzyme
          d- all of the above
          PDF created with pdfFactory Pro trial version www.pdffactory.com
          283- the temperature of the blood refrigerator is:
          a- 6 – 9C ْ
          b- 2 – 6 C ْ
          c- 9 – 15 C ْ
          d- 20 – 25 C ْ
          284- biochemical changes in the stored blood RBCs:
          a- RBCs change it’s shape
          b- osmotic fragility is increased
          c- loss of red cell membrane lipids
          d- all of the above
          285- the red cells in frozen state can be stored for years by addition of:
          a- oxygen
          b- hydrogen
          c- nitrogen
          d- glycerol
          286- dark red card on blood bag means:
          a- group O +ve
          b- group A +ve
          c- group B +ve
          d- group AB +ve
          287- panel test is done to know:
          a- viruses on the serum
          b- antigen on the serum
          c- antibodies on the serum
          d- all of the above
          288- incomplete antibody :
          a- IgM
          b- IgA
          c- Ig G
          d- Ig D
          PDF created with pdfFactory Pro trial version www.pdffactory.com
          289- the anticoagulant used for blood coagulation and platelet function
          studies:
          a- EDTA
          b- heparin
          c- trisodium citrate
          d-all of the above
          290- the preferred anticoagulant for blood cell count :
          a- EDTA
          b- heparin
          c- trisodium citrate
          d-all of the above
          291- haymen’s solution is used for:
          a- RBCs counting
          b-WBCs counting
          c- platelet counting
          d- Hb concentration
          292-the most commonly used diluting fluid for RBCs count:
          a- Drakin’s solution
          b- Turk’s solution
          c- Normal saline
          d- glacial acetic acid
          293- Drabkin’s solution is used for:
          a- RBCs counting
          b-WBCs counting
          c- platelet counting
          d- Hb concentration
          294-in hemoglobin determination by using cyanmethemoglobin method . we
          measure the absorbency in spectrophotometer at a wave length of:
          a- 504 nm
          b- 405 nm
          c- 540 nm
          d- 450 nm
          PDF created with pdfFactory Pro trial version www.pdffactory.com
          295- the ratio of the volume of cellular elements to that of whole blood is:
          a- MCV
          b-MHC
          c- MCHC
          d- HCT
          296- the normal value of mean corpuscular volume is:
          a- 27 – 32 pg
          b- 80 – 90 fl
          c- 33 – 38%
          d- non of the above
          297- (PCV/ RBCs count ) x 10 =
          a-MCH
          b- MCHC
          c-MCV
          d- HCT
          298- thoma pipette is used for :
          a- RBCs counting
          b-WBCs counting
          c- platelet counting
          d- Hb concentration
          299- glacial acetic acid is used for :
          a- RBCs counting
          b-WBCs counting
          c- platelet counting
          d- reticulocyte counting
          300- for counting platelet we use:
          a- ammonium oxalate
          b- mercury chloride
          c- sodium carbonate
          d- ammonium hydroxide
          PDF created with pdfFactory Pro trial version www.pdffactory.com
          301- normal adult have a reticulocyte count of :
          a- 2.5 % - 6.5 %
          b- 0.5 % - 2%
          c- 5 % - 10%
          d- non of the above
          302- brilliant cresyl blue is used for:
          a- RBCs counting
          b-WBCs counting
          c- platelet counting
          d- reticulocyte counting
          303- Westergren’s pipette is used for :
          a- HCT estimation
          b- ESR estimation
          c- Hb determination
          d-WBCs counting
          304-Lowenstein-Jensen media is used for the isolation for:
          a- neisseria gonorrhea
          b- mycobacterium tuberculosis
          c- haemophilus influenza
          d- staphylococcus aureus
          305- Hekton-Enteric agar is the selective media for:
          a- salmonella
          b- streptococcus
          c- staphylococcus
          d- all of the above
          306- the selective media for isolation of fungi is:
          a- S-S agar
          b- XLD agar
          c- sabouraud glucose agar
          d- Hekton-Enteric agar
          PDF created with pdfFactory Pro trial version www.pdffactory.com
          307- gram positive rods with Chinese letter appearance:
          a- mycobacterium tuberculosis
          b- corynebacteria diphtheria
          c- clostridium tetani
          d- staphylococcus pneumonia
          308- E lek test is done to diagnose :
          a- streptococcus
          b- staphylococcus aureus
          c- clostridium tetani
          d- corynebacteria diphtheria
          309- all are lactose fermenter except:
          a- E-COLI
          b- proteus
          c- klebsiella
          d- enterobacter
          310- produce pale colonies onMacConkey’s agar and have tendency to
          swarm on blood agar:
          a- salmonella
          b- shigella
          c- klebsiella
          d- proteus
          311- lactose frementer gram negative bacilli with mucoid growth:
          a- salmonella
          b- shigella
          c- klebsiella
          d- proteus
          312-non lactose fermenter gram negative bacilli produce H2S :
          a- salmonella
          b- shigella
          c- klebsiella
          d- proteus
          PDF created with pdfFactory Pro trial version www.pdffactory.com
          313- Widal test is done for diagnosis of :
          a- salmonella
          b- shigella
          c- E-coli
          d- klebsiella
          314- the causative agent of enteric fever:
          a- salmonella
          b- shigella
          c- klebsiella
          d- proteus
          315- the most common cause of urinary tract infection :
          a- E-coli
          b- salmonella
          c- shigella
          d- streptococcus
          316- an important cause of diarrhea in infant:
          a- staphylococcus
          b- E-coli
          c- salmonella
          d- shigella
          317- the most common causative agent for peptic ulcer :
          a- campylobacter
          b- H-pylori
          c- V-cholera
          d- all of the above
          318- TCBS is the selective media for isolation of:
          a- H-pylori
          b- V-cholera
          c- E-coli
          d- H influenza
          PDF created with pdfFactory Pro trial version www.pdffactory.com
          319- gram negative bacilli strict aerobes grows on simple media producing a
          characteristic greenish pigment:
          a- campylobacter
          b- pseudomonas
          c- pasterulla
          d- bordetella
          320- the bacteria which cause scarlet fever:
          a- staphylococcus
          b- streptococcus
          c- salmonella
          d- shigella
          321- antistreptolysin O titer (ASO) is done for the diagnosis of:
          a- group A streptococcus
          b- group B streptococcus
          c- staphylococcus aureus
          d- staphylococcus albus
          323- Loffler’s serum is used for isolation of:
          a- anthrax
          b- clostridium
          c- diphtheria
          d- T.B
          324- gas gangrene is caused by:
          a- clostridium tetani
          b- clostridium botulinum
          c- clostridium welchii
          d- non of the above
          325- Bacillary dysentery is caused by:
          a- salmonella
          b- shigella
          c- cholera
          d- all of the above
          PDF created with pdfFactory Pro trial version www.pdffactory.com
          326- the cause of plague:
          a- Y-enterocolitica
          b- Y-pestis
          c- Y pseudotuberculsois
          d- non of the above
          327- treponema palladium is the cause of :
          a- T.B
          b- gonorrhea
          c- syphilis
          d- AIDS
          328- the venereal disease research laboratory test (VDRL|) is done for
          diagnosis of:
          a- T.B
          b- gonorrhea
          c- syphilis
          d- AIDS
          329- Trachoma is caused by:
          a- mycoplasma
          b- chlamydiae
          c-richettsia
          d- mycobacteria
          330- ………….. are substance produced by specialized cells of the body and
          carried by blood stream where it affect other specialized cells:
          a- vitamins
          b- enzymes
          c- isoenzyme
          d- hormones
          331- ……………is a protein which catalyse one or more specific biochemical
          reaction and not consumed during the reaction:
          a- enzymes
          b- hormones
          c- vitamins
          d- proteins
          PDF created with pdfFactory Pro trial version www.pdffactory.com
          332- …………is required in the hepatic synthesis of prothrombin and the
          blood clotting factors and it’s deficiency is observed in newborn infants:
          a- vitamin E
          b- vitamin A
          c- vitamin K
          d- vitamin D
          333- there are enzymes that catalyze the same reaction but differ in there
          physical properties:
          a- vitamins
          b- adjuvents
          c- isoenzyme
          d- hormones
          334- it’s functionis to maintain adequate serum level of calcium:
          a- vitamin E
          b- vitamin A
          c- vitamin K
          d- vitamin D
          335-the inhibitor and substrate bind at different sites on the enzyme this type
          of inhibition is called:
          a- competitive inhibition
          b- non competitive inhibition
          c- surface recognition
          d- product concentration
          336- ……….. found in cartilage consist of a core protein to which the linear
          carbohydrates chain are covalently attached:
          a- glycoprotein
          b- proteoglycan
          c- link protein
          d- hyaluronic acid
          337- ……………is synthesized only by micro-organism , it’s not present in
          plants but present in liver , it’s deficiency leads to pernicious anemia:
          a- vit C
          b- vit B12
          c-vit B1
          d- vit B2
          PDF created with pdfFactory Pro trial version www.pdffactory.com
          338- the brown color of the stool is due to the presence of:
          a- urobilinogen
          b- urobilin
          c- porphyrin
          d- bilirubin
          339- the degradation of heme takes place in the…………. particularly in the
          liver and spleen
          a- reticulocytes
          b- erythrocytes
          c- reticuloendothelial cells
          d- non of the above
          340- increased Hb destruction , the liver is unable to cup the greater load of
          pigment and bilirubin level well rises this is called :
          a- hepatogenous jaundice
          b- hemolytic jaundice
          c- obstructive jaundice
          d- non of the above
          341- …………plays a role in visual cycle it’s deficiency leads to night
          blindness, β-carotene is the major precursor of this vitamin in human:
          a- vitamin E
          b- vitamin A
          c- vitamin K
          d- vitamin D
          342- a large percentage of the……….. requirement in humans is supplied by
          intestinal bacteria:
          a- biotin
          b- niacin
          c- folic acid
          d- thiamin
          343- ………….. are organic compounds required by the body in trace
          amount , can’t be synthesized by humans , but supplied by diet:
          a- enzymes
          b- vitamins
          c- hormones
          d- proteins
          PDF created with pdfFactory Pro trial version www.pdffactory.com
          344- it facilitate the absorption of iron by reducing it to ferrus state in the
          stomach and it’s deficiency result in scurvy:
          a- vit C
          b- vit B
          c- vit D
          d- vit A
          345- it’s function is to transport oxygen from the lung to the tissue:
          a- haptoglobin
          b- hemoglobin
          c- bilirubin
          d- myoglobin
          346- ………….. act as an antioxidant and it’s deficiency cause liver
          degeneration:
          a- vit E
          b- vit A
          c- vit K
          d- vit D
          347- it’s caused by liver parenchyma damage , the excretion of bile greatly
          decreased and the concentration of bilirubin in the blood rise :
          a- hemolytic jaundice
          b- hepatogenous jaundice
          c- obstructive jaundice
          d- non of the above
          348- ………play an essential role in body metabolism , a deficiency or excess
          may lead to serious dearrangement of body function:
          a- enzymes
          b- hormones
          c- vitamins
          d- isoenzymes
          349- …………. will interfere with the chemical determination of bilirubin ,
          giving high variable results:
          a- hemolysis
          b- hemoglobin
          c- vitamins
          d- hormones
          PDF created with pdfFactory Pro trial version www.pdffactory.com
          350- in hemolytic jaundice there will be increase……………. in serum:
          a- direct bilirubin
          b- indirect bilirubin
          c- total bilirubin
          d- all of the above
          351- the inhibitor binds reversibly to the same site on the enzyme that the
          substrate normally occupy , this type of inhibition is called :
          a- competitive inhibition
          b- non competitive inhibition
          c- surface recognition
          d- product concentration
          352- regulation of blood glucose level can be achieved by :
          a- hormonal mechanism
          b- hepatic and renal mechanism
          c- (a) & (b)
          d- non of the above
          353- insulin is a hormone secretes by :
          a- α cell of islet of langerhans in pancreas
          b- β cell of islet of langerhans in pancreas
          c- suprarenal cortex
          d- non of the above
          354- cholesterol is a component of all cell membrane and it’s the precursor
          of :
          a- bile acid
          b- steroid hormones
          c- vit D
          d- all of the above
          355- anti diuretic hormone ( ADH) secretion is controlled by:
          a- rennin angiotensin
          b- plasma osmlality
          c- (a) & (b)
          d- non of the above
          PDF created with pdfFactory Pro trial version www.pdffactory.com
          356- the electrophoresis is based on differential migration of :
          a- charged particles
          b- uncharged particles
          c- molecular weight
          d- (a) & (b)
          357- acid base balance is regulated by :
          a- oxygen concentration
          b- hydrogen ion concentration
          c- nitrogen ion concentration
          d- (a) & (b)
          358- it’s an increase in hydrogen ion concentration of the blood:
          a- acidosis
          b- alkalosis
          c- acid base balance
          d- (a) & (b)
          359- over production of acid associated with :
          a- diabetes mellitus
          b- lactic acidosis
          c- methanol poisoning
          d- all of the above
          360- serum bicarbonate is decreased in:
          a- respiratory acidosis
          b- metabolic acidosis
          c- renal tubular acidosis
          d- all of the above
          361- chronic deficiency in dietry calcium can lead to :
          a- anemia
          b- bronchial asthma
          c- osteoporosis
          d- non of the above
          PDF created with pdfFactory Pro trial version www.pdffactory.com
          362- …………is due to decrease blood CO2:
          a- metabolic acidosis
          b- respiratory acidosis
          c- respiratory alkalosis
          d- metabolic acidosis
          363- …………. is the most important factor affecting body sodium content:
          a- aldosteron secretion
          b- antidiuretic hormone
          c- testosterone
          d- all of the above
          364- haemosiderosis is :
          a- increase iron store
          b- decrease iron store
          c- increase hemoglobin
          d- decrease hemoglobin
          365- if there’s a mixture of protein ( colloids) and salt( crystalloid) they can
          be separated by :
          a- precipitation
          b- dialysis
          c- chromatography
          d- electrophoresis
          366- the predominant cation in intracellular fluid is :
          a- sodium
          b- potassium
          c- calcium
          d- phosphorus
          367- metabolic acidosis is due to :
          a- failure to secret acid
          b- bronchial asthma
          c- loss of bicarbonate
          d- (a) &(c)
          PDF created with pdfFactory Pro trial version www.pdffactory.com
          368- high level of plasma ferritin may occur due to :
          a- inflammatory condition
          b- malignant disease
          c- liver disease
          d- all of the above
          369- gonadal hormones estimation is important in :
          a- detection of ovulation
          b- assessment of amenorrhea
          c- evaluation of delayed puberty
          d- all of the above
          370- the secretion of gonadal hormone is controlled by :
          a- LH
          b- FSH
          c- TSH
          d- (a) & (b)
          371- the intensity of the color is directly proportional to the………. of the
          analyte in the solution:
          a- dilution
          b- contamination
          c- concentration
          d- observation
          372- the…………contain information of any health or safety rich associated
          with use or exposure to hazardous chemicals:
          a- MSDS
          b- NFPA
          c- POLT
          d- OSHA
          373- instraument used to measure color changes in the labs:
          a- microscope
          b- centrifuge
          c- photometer
          d- all of the above
          PDF created with pdfFactory Pro trial version www.pdffactory.com
          374- the color coded signs used to identify flammable chemicals:
          a- blue
          b- yellow
          c- white
          d- red
          375- quality assurance includes :
          a- personal orientation
          b- laboratory documentation
          c- knowledge of laboratory istraumentation
          d- all of the above
          376- the laboratory procedure manual include:
          a- patient preparation
          b- specimen collection & processing
          c- specimen preservation , storage & transport
          d- all of the above
          377- the principal of reflectance photometer
          a- measure the amount of light that pass through the solution
          b- measure the amount of light that the solution absorbs
          c- (a) & (b)
          d- non of the above
          378- the blood cell counter include :
          a- aperture impedence cell counter
          b- Geiger counter
          c- microscopes
          d- all of the above
          379- it is mession is to save lives , prevent injuries , and protect health of all
          workers in the lab. :
          a- MSDS
          b- NFPA
          c- POLT
          d- OSHA
          PDF created with pdfFactory Pro trial version www.pdffactory.com
          380-……………. requires 3 hours at 140 Cْ or 1 hour at 160 C ْ for complete
          sterilization
          a- hot air oven
          b- autoclave
          c- filteration
          d- all of the above
          381- arterial blood samples are essential to do :
          a- CBC
          b- urea
          c-blood glucose
          d- blood gas analysis
          382- vaccum tubes with green stopper contain:
          a- EDTA
          b- sodium citrate
          c- heparin
          c- no anticoagulant
          383- serum separator tube is all of the following except:
          a- contain gel that separate serum from cells during centrifugation
          b- contain clot activator to speed clot formation
          c- has red & black mottled top stopper
          d- used for coagulation tests
          384- for phlepotomy we use all of the following except:
          a- the hypodermic needle & syringe
          b- the vaccum tube system
          c- the monolet lancets
          d- the winged infusion set
          385- lab. equipment should be cleaned and disinfected with :
          a- hypochlorite
          b- formaldehyde
          c- glutaraldehyde
          d- (b) & (c)
          PDF created with pdfFactory Pro trial version www.pdffactory.com
          386- any blood split in the lab should be immediately swabbed with :
          a- hypochlorite
          b- alcohol
          c- soap
          d- water
          387- CBC is performed using:
          a- serum
          b- well mixed EDTA whole blood
          c- plasma
          d- non of the above
          388- which tube should be filled first in blood collection:
          a- tubes with anticoagulant
          b- tube without anticoagulant
          c- tubes for blood culture
          d- non of the above
          389- the monojector is designed to be used with :
          a- the monolet lancet
          b- tenderlett
          c- tenderfoot
          d- non of the above
          390- the site of choice for capillary puncture in newborns is :
          a- the earlobe
          b- middle finger
          c- the big toe
          d- the lateral medial planter heel surface
          391- M-tuberculsis bacilli stain with :
          a- gram stain
          b- Zheil Nelson stain
          c- Gimesa stain
          d- all of the above
          PDF created with pdfFactory Pro trial version www.pdffactory.com
          392- AIDS is transmitted through :
          a- food
          b- blood
          c- semen
          d- (b) & (c)
          393- bacteria which cause syphilis:
          a- Neisseria gonorrhea
          b- Viencent angina
          c- Treponema palladium
          d- Yersinia pestis
          394- to diagnose syphilis:
          a- RPR
          b- VDRL
          c-Wasserman
          d- all of the above
          395- streptococci secret:
          a- streptolysinO
          b- streptolysin S
          c- streptokinase
          d- all of the above
          396- disease caused by streptococci:
          a- scarlet fever
          b- purperal sepsis
          c- rheumatic fever
          d- all of the above
          397- staphylococci secrets:
          a- coagulase enzyme
          b- fibrinolysin
          c- hyaluronidase
          d- all of the above
          PDF created with pdfFactory Pro trial version www.pdffactory.com
          398- gram positive bacilli:
          a- Klebsilla
          b- Salmonella
          c- Proteus
          d- C-diphtheria
          399- meningeococcal meningitis is transmitted by :
          a- food
          b- droplet
          c- touch
          d- all of the above
          400- dark field microscopy is used to diagnose :
          a- T.B.
          b- syphilis
          c- gonorrhea
          d- AIDS
          401- it cause food poisining with flacid paralysis:
          a- clostridium tetani
          b- clostridium welchii
          c- clostridium botulinium
          d- all of the above
          402- the infective stage of plasmodium vivax :
          a-merozoites
          b- sporozoites
          c- schizont
          d- trophozoite
          403- Pirenella conica snail is the intermediate host of:
          a- schistosoma haematobium
          b- fasciola hiptica
          c- heterphyes heterophyes
          d- diphyllobothrium latum
          PDF created with pdfFactory Pro trial version www.pdffactory.com
          404- Bulinus truncates snail is the intermediate host of :
          a- fasciola hepatica
          b- fasciola gigantica
          c- schistosoma haematobium
          d- schistosoma mansonii
          405- to isolate meningiococci we have to culture the sample on:
          a- Bordet Gengou
          b- modified Thayer martin media
          c- Lowenstein Jensen media
          d- all of the above
          406- to isolate fungi :
          a- Brain-Heart infusion media
          b- tissue culture
          c- Lowenstein –Jensen media
          d- chocolate agar
          407- to isolate H- influenza:
          a- blood agar
          b- chocolate agar
          c- mac Conkey media
          d- all of the above
          408- the bacteria which cause pseudomembrainous conjunctivitis :
          a- N.gonorrhea
          b- C. diphtheria
          c- staphylococcus
          d- Chlamydia
          409- the best sample to diagnose meningitis :
          a- blood
          b- sputum
          c- CSF
          d- urine
          PDF created with pdfFactory Pro trial version www.pdffactory.com
          410- used to stain Chlamydia
          a- gram stain
          b- giemsa stain
          c- wright stain
          d- all of the above
          411-……….. is used as transport medium for sample in which cholera is
          suspected
          a- Cary-Blair media
          b- Stuart media
          c- Alkaline peptone water
          d- glycerol
          412- the color of XLD medium:
          a- green
          b- red
          c- yellow
          d- blue
          413- CIN medium is used to isolate:
          a- E.coli
          b- Vibrio cholera
          c- yersinia
          d- salmonella
          414- to make wet mount preparation:
          a- 10 % KOH
          b- 10 % Na OH
          c- 10 % Na CO3
          d- 10% Na Cl
          415- we do wet mount preparation for vaginal smears to diagnose:
          a- T.vaginalis
          b- N.gonorrhea
          c- streptococci
          d- staphylococci
          PDF created with pdfFactory Pro trial version www.pdffactory.com
          416- to isolate viruses:
          a- Loeffler media
          b- tissue culture
          c- Bordet –Gengou media
          d- Brain- Heart infusion
          417- to diagnose whooping cough :
          a- Bordet –Gengou media
          b- Lowenstein –Jensen media
          c- modified Thayer martin media
          d- New York city agar
          418- to diagnose systemic infection we do :
          a- urine culture
          b- CSF culture
          c- blood culture
          d- sputum culture
          419- we give no growth for blood culture after:
          a- 1 week
          b- 8 weeks
          c- 6 weeks
          d- 3 weeks
          420- to dissolve mucous in sputum sample :
          a- 10% NaOH
          b- 30 % NaOH
          c- 10 % KOH
          d- 10% NaCl
          421- mutualism means:
          a- one partener benefits , other unaffected
          b- both partner benefit
          c- one partner benefit , other damaged
          d- living together
          PDF created with pdfFactory Pro trial version www.pdffactory.com
          422- Commensalisms means:
          a- living together
          b- one partner benefit , other damaged
          c- both partner benefit
          d- one partner benefits , other unaffected
          423- Balantidium coli moves by:
          a- flagella
          b- cilia
          d- pseudopod
          d- all of the above
          424- Mouth inhabitant:
          a- Trichomonas hominis
          b- Trichomonas tenax
          c- Trichomonas vaginalis
          d- giardia lamblia
          425- Transmitted by sexual intercourse:
          a- toxoplasma
          b- giardia lamblia
          c- Trichomonas vaginalis
          d- all of the above
          426- Intermediate host of Trypansom:
          a- triatoma megista
          b- sand fly
          c- tse tse fly
          d- anopheles
          427- The cause of chaga's disease:
          a- trypanosoma gambiense
          b- trypansoma rhodesiense
          c- trypansoma cruzi
          d- leishmania braziliense
          PDF created with pdfFactory Pro trial version www.pdffactory.com
          428- The cause of sleeping sickness:
          a- trypanosoma gambiense
          b- trypanosoma cruzi
          c- trypanosoma rhodesiense
          d- (a) & (c)
          429- Cause Kala- azar:
          a- leishmania tropica
          b- leishmania braziliense
          c- leishmania donovani
          d- leishmania mexicana
          430-cause oriental sore:
          a- plasmodium ovale
          b- leishmania tropica
          c- leishmania donovani
          d- trypanosoma rhodesiense
          431- Its trophozite is shaped like a pear , has the 2 nuclei that resembles eyes
          and 4 pairs of flagella that look like hair:
          a- Trichomonas vaginalis
          b- entameoba histolytica
          c- giardia lamblia
          d- endolimax nana
          432- Pear shaped trophozite with 4 anterior flagella and a 5th forming the
          outer edge of a short undulating membrane:
          a- Trichomonas hominis
          b- entameoba histolytica
          c- entameoba coli
          d- endolimax nana
          433- sometimes it cause metastatic infection which involve liver, lung, brain
          or other viscera:
          a- giardia lamblia
          b- Trichomonas vaginalis
          c- entameoba histolytica
          d- balantidium coli
          PDF created with pdfFactory Pro trial version www.pdffactory.com
          434- Intestinal ciliate:
          a- entameoba histolytica
          b- entameoba coli
          c- giardia lamblia
          d- balantidium coli
          435- Asexual multiplication of plasmodium vivax takes place in:
          a- anopheles
          b- sand fly
          c- human
          c- tse tse fly
          436- Plasmodium falciparam is transmitted by :
          a- triatoma megista
          b- tse tse fly
          c- anopheles
          d- sand fly
          437- Moves by pseudopods:
          a- giardia lamblia
          b- balantidium coli
          c- entameoba histolytica
          d- Trichomonas vaginalis
          438- it's one of the round worms:
          a- schistosoma mansoni
          b- schistosoma haematobium
          c- ascaris lumbricoides
          d- fasciola hepatica
          439- it's one of the tape worms:
          a- ascaris lumbricoides
          b- ancylostoma duodenal
          c- trichuris tricura
          d- taenia saginata
          PDF created with pdfFactory Pro trial version www.pdffactory.com
          440- barrel shapped egg, yellow brown in color with a colorless protruding
          mucoid plug in each end:
          a- egg of ascaris lumbricoides
          b- egg of ancylostoma duodenal
          c- egg of trichuris tricura
          d- egg of taenia saginata
          441- large oval egg , pale yellow brown in color has a characteristic side spine
          and contain a fully developed miracidium , the worm is:
          a- S.mansoni
          b- S. hematobium
          c- A. duodenal
          d- T. solium
          442- large oval egg , pale yellow brown in color has an indistinct operculum
          and contains unsegmented ovum:
          a- S. hematobium
          b- fasciola hepatica
          c- heterophyes heterophyes
          d- taenia solium
          443- round egg , embryo is surrounded by a thick brown radially striated
          wall , hooklets are present in the embryo:
          a- S. hematobium
          b- fasciola hepatica
          c- A. duodenal
          d- T. solium
          444- oval colorless egg , flattened on one side and contains a larvae:
          a- hymenelopis diminuta
          b- dipylidium caninum
          c- entrobius vermicularis
          d- taenia saginata
          445- the cause of malignant malaria:
          a- plasmodium vivax
          b- plasmodium ovale
          c- plasmodium malaria
          d- plasmodium falciparum
          PDF created with pdfFactory Pro trial version www.pdffactory.com
          446- infection occur when infective filariform larvae penetrate the skin:
          a- ascaris lumbricoides
          b- ancylostoma duodenal
          c- fasciola hepatica
          d- heterophyes heterophyes
          447-……..lives in the liver and bile ducts of sheep and cattle:
          a- stronyloides stercoralis
          b- schistosoma mansoni
          c- fasciola hepatica
          d- ancylostoma duodenal
          448- segment found in stool which is white and opaque and measures 20mm
          long by 6mm wide with uterus that has a central stem and more than 13 side
          branches on each side…the worm is :
          a- fasciola hepatica
          b- trichuris trichuris
          c- heterophyes heterophyes
          d- taenia saginata
          449- infection is by eating raw or under cooked fish:
          a- fasciola hepatica
          b- trichuris trichuris
          c- heterophyes heterophyes
          d- taenia solium
          450-…………is transmitted by eating raw or under cooked beef:
          a- heterophyes heterophyes
          b- taenia saginata
          c- schistosoma mansoni
          d- ancylostoma duodenal
          451- urine output < 400 ml/24 hours is :
          a- polyuria
          b- anuria
          c- oligouria
          d- non of the above
          PDF created with pdfFactory Pro trial version www.pdffactory.com
          452- precipitation of urates takes place in:
          a- acidic urine
          b- alkaline urine
          c- neutral urine
          d- all of the above
          453- it's one of the causes of persistently acidic urine:
          a- urinary tract infection
          b- phenylketonurea
          c- excessive bicarbonate ingestion
          d- excessive ingestion of soda
          454- common cause of proteinuria:
          a- alcoholism
          b- fasting > 18 hours
          c- diabetes mellitus
          d- Bence- Jones proteins
          455- dipstick detect acetoacetic acid and acetone which react with:
          a- peroxides
          b- nitroprusside
          c- diazo compounds
          d- indoxyl esters
          456- in dipstick bilirubin reacts with :
          a- nitroprusside
          b- peroxides
          c- indoxyl esters
          d- diazo compounds
          457- among the common cause of hematouria:
          a- urogenital carcinoma
          b- diabetes mellitus
          c- heavy exercise
          d- metabolic disorder
          PDF created with pdfFactory Pro trial version www.pdffactory.com
          458- large number of hyaline cast indicated:
          a- acute pyelonephritis
          b- proliferative glomerulonephritis
          c- heart failure
          d- all of the above
          459- red cell casts indicates:
          a- acute pyelonephritis
          b- proliferative glomerulonephritis
          c- heart failure
          d- all of the above
          460- crystals which look like envelope :
          a- triple phosphate
          b- cystine
          c- uric acid
          d- calcium oxalate
          461- Biuret test is done to determine:
          a- glucose
          b- pentose
          c- protein
          d- galactose
          462- the 1st tube of synovial fluid is for:
          a- hematology
          b- chemistry
          c- microbiology
          d- microscopy
          463- square plate like crystals with notched corners in synovial fluid indicate:
          a- uric acid
          b- calcium pyrophosphate
          c- cholesterol
          d- monosodium urate
          PDF created with pdfFactory Pro trial version www.pdffactory.com
          464- abnormal forms in semen should not exceed:
          a- 10 %
          b- 5 %
          c- 25 %
          d- 50 %
          465- citrate utilization test is done to assist identification of:
          a- gram +ve bacteria
          b- gram –ve bacteria
          c- entrobacteria
          d- enterococcus
          466- the Kovac's reagent used in the following biochemical tests:
          a- catalase
          b- coagulase
          c- indole
          d- methyl red
          467- positive results for H2S production appear as ………. Colour:
          a- black
          c- yellow
          c- red
          d- green
          468-………. Give positive coagulase test:
          a- streptococci
          b- staphylococcus aureus
          c- staphylococcus saprophyticus
          d- Escherichia coli
          469- methyl red test is performed with:
          a- Erlich reagent
          b- Kovac's reagent
          c- Voges proskaur
          d- non of the above
          PDF created with pdfFactory Pro trial version www.pdffactory.com
          470-…………give positive result with urease test:
          a- salmonella
          b- shigella
          c- Y. enterocolitica
          d- all of the above
          471-……….. test is used to differentiate between bacteroides and brucella:
          a- indole
          b- methyl red
          c- H2S production
          d- nitrate reduction
          472- DNAase test is positive with:
          a- streptococcus pneumonia
          b- E.coli
          c- staphylococcus aureus
          d- staphylococcus epidermis
          473-………… solution used in the gram stain technique acts as a mordant:
          a- crystal violet
          b- safranine
          c- iodine
          d- alcohol
          474- the counter stain in Ziehl- Neelson stain is :
          a- malachite green
          b- methylene blue
          c- iodine
          d- (a) & (b)
          475- bile solubility test is positive with :
          a- streptococcus viridans
          b- streptococcus pneumonia
          c- streptococcus agalectiae
          d- streptococcus pyrogenes
          PDF created with pdfFactory Pro trial version www.pdffactory.com
          476-litmus milk decolorization test assist the identification of :
          a- entrobacteria
          b- bacteroides
          c- brucella
          d- enterococci
          477-The process of getting things done through and with people operating in
          organized group toward a common goal is the
          a- management
          b- Organization
          c- Planning
          d- None of the above
          478- Primary objectives in the planning are directed to
          a- the laboratory as a whole
          b- Increase the efficiency in the performance of the lab. test
          c- Decrease the costs in the performance of the lab. test
          d- All of the above
          479- Forecasting needs for staff personnel means
          a-Prediction in relation to the kind of technician and technologist who will
          be working in the lab.
          b- Plan for the full utilization of efficient use of instrument
          c- Plan for the full use of space in the lab.
          d- None of the above
          480-An organization
          a- Is formed when 2 or more persons are brought together to achieve a
          common goal
          b- Is closely related to planning
          c- Involves structuring activities and functions within institutions to
          achieve the goals and objects
          d- all of the above
          481- The real behavior and relationships of organization members usually
          differ from their planned behavior and relationships. It is
          a- Formal organization
          b- Informal organization
          c- Space utilization
          PDF created with pdfFactory Pro trial version www.pdffactory.com
          d- None of the above
          482- The intra lab. System includes the following except
          a- Calendar format
          b- Histogram format
          c- out of limits report sheet
          d- Proficiency testing and computer program
          483- The out of limits report form provides
          a- Space for recording reagents changes
          b- Control lot number changes
          c- Serve as a general "dairy" of the test methodology
          d- All of the above
          484- Patient preparation, specimen collection and technical performance of
          lab. test are general categories of…………..
          a- Planning
          b- Utilization of space
          c-Work flow
          d- Quality control
          485-floor book manual includes the following except
          a- Test name
          b- Sample fluid
          c-Minimum volume
          d- Proper procedures for collecting routine and special tests
          486- Collection procedure manual involve
          a- Blood collection from pediatric patients
          b- Intensive care blood collection
          c- Isolation techniques for lab. Personnel
          d- All of the above
          487-On the container and \or lab requisition
          a- Patient's full name should be put
          b- Hospital number should be put
          c- Date of collection should be put
          d- All of the above
          PDF created with pdfFactory Pro trial version www.pdffactory.com
          488 Accuracy referred to the following except
          a- Correctness and exactness of the test
          b- Closeness of the test to the true value
          c- True value determined by comparison to a standard
          d- reproducibility
          489- Regarding precision the following is true except
          a- Reproducibility
          b- Closeness of the test results to one another when using the same specimen
          c- In the clinical lab it is expressed as (SD) and coefficient of variation
          d- The capability of the method to detect a small amount of substance with
          some assurance
          490- Reliability is
          a- The ability of a method to measure only that substance being tested
          b- The ability of the test method to maintain its accuracy despite of
          extraneous circumstances
          c- The ability of the method to maintain accuracy, precision and ruggedness
          d- None of the above
          491- - …………….. This symbol in the flow chart means
          a- Beginning process
          b- Decision
          c- Manual operation
          d- Decision mod
          492- This symbol in the flow chart means
          a-Beginning process
          b- Decision
          c- Direction flow
          d- Document
          493- Work load on which personnel requirements are usually based is
          influenced by
          a- changes in volume
          b- Test mix
          c- Patient population
          PDF created with pdfFactory Pro trial version www.pdffactory.com
          d- All of the above
          494- The physical features of the lab. one of the measures of
          a- forecasting of personnel needs
          b- Assessment of space utilization
          c- Time management
          d- None of the above
          495- If P (E) is the probability of E we may express this definition as
          a- P (E) = m\N
          b- P (E) = N\m
          c- P (E) = m X N
          d- None of the above
          496- When a test indicates a positive status when the true status is negative it
          is called
          a- positive test
          b- False positive test
          c- Negative test
          d- False negative test
          497- The specificity of a test
          a- The probability of a positive test results or (presence of the symptoms)
          given the presence of the disease
          b- The probability of a negative test results or (absence of the symptoms)
          given the absence of the disease
          c- a and b
          d-None of the above
          498- The largest collection of entities for which we have an interest at a
          particular time is called
          a- Population
          b- Sample
          c- Data
          d-All of the above
          499- A sample is
          a- A part of a population
          b- The whole population
          c- Endless population
          PDF created with pdfFactory Pro trial version www.pdffactory.com
          d- None of the above
          500- If we have 100 students and they are ranked by age beginning with the
          4th student, every tenth student is chosen (the student no. 4 then 14 and 24
          and so on) this type of sample is called
          a- Systemically selected sample
          b- A stratified selected sample
          c- Simple random sample
          d- Cluster selected sample
          501-A point estimate is
          a- A single numerical value used to estimate the corresponding population
          parameter
          b- Tow numerical values defining a range of values include the parameter
          being estimated
          c- a and b
          d- None of the above
          502-A statistical inference is
          a- A procedure by which we reach a conclusion about population based on the
          information obtained from the sample drawn from it
          b- The cause behind estimation in the health science fields
          c- Calculated data from the data of the sample that are approximation of
          the corresponding parameter
          d- None of the above
          503- Estimator is
          a- the rule that tells us how to compute the single value which is called
          estimate
          b- Two numerical values defining the range of values
          c- a, b
          d- None of the above
          504 The table which shows the way in which values of the variables are
          distributed among the specified class interval is called
          a- Relative frequency
          b- Ordered array
          PDF created with pdfFactory Pro trial version www.pdffactory.com
          c- Frequency table
          c- None of the above
          505- The following are the ages of 5 patients seen in the emergency room in
          certain day 35, 30, 55, 40, 25 years the mean of their ages is
          a- 37 years
          b- 30 years
          c- 39 years
          d- 40 years
          506- A mathematical tool designed to facilitate complex clinical decision in
          which many variables must be considered spontaneously is called
          a- Reference value
          b- Decision analysis
          c- Quality assurance
          d- None of the above
          PDF created with pdfFactory Pro trial version www.pdffactory.com

          تعليق


          • #6
            تابع:

            IV-هذا من شخص آخر :

            موقع مختبرات العرب
            http://www.arabslab.com/vb/showthread.php?t=12659
            هذه أسئلة و لكن الإجابات غير مؤكدة لأنها إجابة طالب :

            اختبآر الهيئـه : 06-10-2009
            اول شي هذا بعض الاسئله اللي جاتني وانا اجتهد وشرحت في بعضها كيف لفو فيها وسامحوني اذا وجدتم اي خطاء

            1) One is not found (involve) in colorimetric:
            1- Cuvett
            2-light source
            3- Photo sensor and analyzer
            4-fule source
            5-Filter
            2) Direct anti-antibody test used to detect:
            1- Sensitized RBCs in patient blood
            2- IgG
            3- IgM
            4- Sensitized antibody in patient serum
            5- Non of the above
            السؤال هذا جاني عكسه يقول في anti-globin واجابته الدايركت في السيرم
            3) Detection of malaria parasite is by:
            1- Thick blood film
            هذا السؤال جاني بطريقه غريبه جدا وحتى الاجابات كانت غريبه بس اللي خلاني افهمه لكشف الملاريا بس هذه الكلمتين اللي فهمتها ومن الاجابه انه مكتوبه في blood film
            4) One is always non motile gram –ve rods:
            1-Haemophilus Influenza 2-Shigella
            3- E.coli 4- Salmonella
            5- Borditella Pertusis

            5) One is motile gram –ve rods:
            1- Haemophilus Influenza 2- Bacillus anthracis
            3- Pseudomonas aerogenes 4- Vibrio cholerae
            5- Yersinia Pestis
            6) Autoinfection of human can caused in case of:
            1- Entrobius Vermicularis 2- Taenia Sp.
            3- Anclystoma Dudenal 4- Ascaris Lumbercoidis
            السؤال هذا جاني بطريقه مدري كيف المهم انها معقده وبصراحه انا عرفتها من الاجابات
            7) One is not true about acid:
            1- Proton donor
            2- Turn litmus paper to blue >> (( true red ))
            3- Sour taste
            4- React with alkaline to give water and salt
            5- Non of the above
            السؤال هذا جاني في الاسد والالكالاين يعني عكس بعض
            8-Blood of Microfilaria infection diagnostic with:
            Wuchereria bancrofti
            9.In Which Substance Give Blood Glucose When Hydrolysis :
            Muscle Glycogen Liver Glycogen
            Heart Glycogen Unsaturated Fat
            Triglyceride
            السؤال هذا جاني بس كيف طريقته يقول ايش waht glycogen when hydrolysis in blood glucoseوفي الاجابات مكتوب العضو يعني كبد لوحده
            10)Hemolytic Anemia In Newbone:
            Mother + Rh
            Father + Rh
            Baby – Rh
            B And C
            All Of The Above
            11)O+Ve Has :
            No Abo Group
            No Rh
            12)in cardiac infraction one of the measurements is not benefited:
            Creatinine
            CK
            LDH
            GOT
            13) regulation of calcium level in serum by:
            Calcitonine hormone only
            Parathyroid hormone only
            Calcionine with parathyroid hormone
            Vitamin D
            PTH+Vitamin D+calcitonin
            14) Syphilis is caused by...
            a. treponema pallidum
            b. neisseria gonorrhea
            c. chlamydia trachomatis
            d. francisella tularensis

            15) VDRL & RPR for
            treponema pallidum
            اخواني اما الاسئله 12 3 4 5 الاجابات نفسها بس ابو التوهاق في الاسئله يبغالكم تركزوا الله يخليكم
            16)The following organisms are lactose fomenters except
            a-E. coli
            b-Enterobacter cloacae
            c-Shigella sonnei
            d-Proteus spp.
            17)The vector for leishmanya infection is :-
            a-Tsetse fly
            b-Sand fly
            c-Housefly
            d-Anopheles mosquito
            e-Reduvid bugs
            18)ALL OF THE FOLLOWING CAN ASSES THE LIVER FUNCTION EXCEPT :-
            A-AST
            B-ALT
            C-ALP
            D-CREATININE
            19_.To detect pH from solution use :
            § pH meter
            § Indicator
            § Micrometer
            20.method used to detect horomone in the laboratory :
            ELIZA
            21.substance used in catalyse reaction :
            H2O2
            22. Normal range of erythrocyte
            - 4,5- 6,5X 10^9
            23-low effective sterilization with:
            Ethyl alcohol 70%
            Methyl alcohol
            chloroform
            5%phenol
            5% cresol
            24-does not sterile with hot air oven:
            Dry glassware
            Oil
            Powder
            Waxes
            Rubber gloves
            25) Hb A2 is consisting of:
            1- 2 ά chains and 2 γ chains
            2- 2 ά chains and 2 β chains
            3- 2 ά chains and 2 δ chains
            4- 2 ά chains and 3 δ chains
            5- 3 ά chains and 2 δ chains
            اما السؤال هذا جاني بنفس الطريقه بس الاختلاف فيه يقول Hb F واللي هو جوابه رقم واحد
            26) Life span of red blood cells in the circulation (body):
            1- 80 days
            2- 120 day
            3- 130 day
            4- 20 day
            5- 7 days


            27) If you see this sign (symbol) in the lab it means:
            1- Flammable
            2- Corrosive
            3- Oxidizing
            4- Explosive
            5- Toxic
            28- org cause bloody in stool:
            Shigella sp.
            وهذا السؤال جاني نصيا مثل ماهو وجاء سؤال مثله بس يقول فيه worm cause blood in urin واجابته shistesoma hematobim

            V-هذا من شخص آخر :

            موقع مختبرات العرب
            http://www.arabslab.com/vb/showthread.php?t=15916
            هذه أسئلة و لكن الإجابات غير مؤكدة لأنها إجابة طالب :

            أسئلة امتحان البرومترك . الثلاثاء 30-3-2010

            All the following can be done by hemocytometer except

            a)RBCs

            b)Semen

            c)Lymphocyte

            d)Total White blood cells



            To fixation of thin blood film we can used

            a) aceton
            b) acitic acid
            c) acid -alchol
            d) Not fixed



            Refrance range of Platelets

            a) 100 x 10 (12)
            b)10 x 10 (9)
            c) 10 x 10 (9)
            d) 10 x 10 (9)



            After collected blood on plain tube must be

            a) no clot

            keep on incubator (37) at 30 min

            c- heamolysis

            d- mix gently



            scotch-tap test used for

            a- ascaris lumbricoides
            b- ancylostoma duodenal
            c- trichuris tricura
            d- taenia saginata

            مو متأكد لأني ما سمعت بالاختبار هذا



            The media can growth some bacteria and inhibition another bacteria is

            A- basic media

            B- diffrentioal media

            C- sellective media

            D- enriched media



            Normal range of Platelets

            a) 150.000 – 450.000*
            b) 250.000 – 500.000
            c) 150 - 400
            d) 20 -80



            Viruses is

            a-Can live by their own

            b-Can not see by light microscope

            c-can not affected by anti-body

            d- can not transport by communicable diseases


            Normal value of Lymphocyte

            a) 55%
            b) 25%
            c) 70%
            d) 10%



            Which of the following of immunoglobulin have five molecules and heavy

            هذا السؤال حيرني كثير

            a_ IgG
            b_ Ig D
            c_ Ig A
            d_igE



            We can different gram –ve bacteria and +ve bacteria differented by

            a)capsule
            b)cytoplasm
            d)cell wall



            antigen In O Positive person

            a) No presence of Ab
            b) No presence of Ag*
            c) No Presence of Ab and Ag



            In which normal presence appear in urin analysis

            a)Protein
            b)Billrubin
            c)Chloride
            d)Glucose



            Rh negative patient means he dose not have

            a) E antigen
            b) D antigen
            c) B antigen
            d) C antigen
            e) Non of the above



            In O blood group the Ab in serum is

            a) Anti A and B**
            b) Anti A
            c) Anti B
            d) Non of the above
            e) All of the above


            it's one of the tape worms
            :
            a- ascaris lumbricoides
            b- ancylostoma duodenal
            c- trichuris tricura
            d- taenia saginata



            All following stained by gimsa stain except

            a)Malaria
            b)Leishmania
            c)Schistosoma
            d) trypanosoma




            . Taenia saginate diagnosis by

            A)Egg in stool

            b)larvae

            C)oocyct



            The unit used in MCV

            A)p/g

            b) ml/ blood

            C)mg/dl

            C)-fl



            Normal range of MCV

            a) 25-60 fl

            b) 75-85 fl

            c) 75-96 fl



            Infective stage of Entrobius Vermicularis is
            :
            1- Larva
            2- Cercaria
            3- Egg
            4- Cyst
            5- Metacercaria



            Immune system can called Adaptive system or

            1- Lympohocyte system
            2-Acquired system
            3- Lympohocyte-immune system


            One is not true about acid:

            1- Proton donor
            2- Turn litmus paper to blue
            3- Sour taste
            4- React with alkaline to give water and salt


            Most common method (technique) used to detect hormone amount in the laboratories
            :
            1- Spectrophotometry
            2- Enzyme Linked Immuno Surbant Assay (ELIZA)


            Strong antigen

            1-vitamin
            2-carbohydrate
            3-proteins


            the Elisa sensitive to light
            :
            1-conjugate only
            2-substrate only
            3-conjugate and substrate



            ESR mesure

            تقاس ب

            1-millimeters
            2-centimeter
            3-miligram


            taenia saginate transmission
            :
            1-ingestion beef
            2-ingestion pork


            Gram positive chain

            1-streptococci
            2-staphyllococci
            3-gonococci
            4-meningococci


            one is gram-motile rod earobic and oxidase possitve

            E.coli

            proteus

            Pseudomonas


            - gondotropic hormone
            LH


            25- Normal prothrombin time (PT) is:

            a- 30-45 seconds
            b- 30-45 minutes
            d- 12-15 minutes
            c : 13+-2 seconds

            بالموجب والسالب يعني ممكن تزيد عن 2 وممكن تنقص


            organism can live and effect urinary tract or vagina

            a- toxoplasma
            b- giardia lamblia
            c- Trichomonas vaginalis
            d- all of the above



            2-lukuocte count become < 4000 in:-
            a-bacteria infection
            b-inflmmation
            c- malignant
            c- non above


            In blood group use:-

            a-know cell with unkown serum
            b- unkown cell with kown serum
            c-know cell with unkown cell


            chinse litter see in :-

            a- C.diftteria b-………. C-……..


            Anigen and Anti-body can bind on

            AG on RBCs and Ab on plasma



            E-coli

            a)motile
            b)indole +ve
            c)lactose vermetive
            d)produce gas and acid
            e)all above



            the result at added by 2 dillution on 6 tube <
            أتوقع صيغة السؤال زي كذا و إجابة السؤال ماني متأكد منها

            a)125
            b)132
            c)164
            d)225

            VI-هذا من شخص آخر :

            موقع مختبرات العرب
            http://www.arabslab.com/vb/showthread.php?t=16701
            http://www.arabslab.com/vb/showthread.php?t=16487
            هذه أسئلة و لكن الإجابات غير مؤكدة لأنها إجابة طالبة :
            1. اسئلة اختباري 25-4-2010
            السلام علييييييييييييييييكم هذي الاسئله الي جتني انا اختبرت في مكة بسم الله نبدأ:

            1-in theplain tube what will happen to the sample:
            clotting.

            2-the unit of measurment of E.S.R:
            millimeter.

            3-the HB which found in 96% is:
            alpha2 beta2.

            4-the most common blood group is:
            O blood group.

            5-the antibody of blood group is:
            antiA and B.

            6-the antibody which found in normal indiviual is:
            antiA and B.

            7-the count of leucocyte is:

            4000-11000x10اس9

            8-the color of anti sera A is:
            blue.

            9-how we can differenite from gram(-)and gram (+) by:
            cell wall.

            10-the color of gram (+) is:
            blue light.

            11-the cell wall of gram(+) contain :
            peptedo glycogen مع ماده ثانيه نسيت اسمها
            12-pemenant defferal :
            A-iron deffeciancy animea.
            B-diarreha.
            C-hepatitis C positive.انا اخترت ذي
            D-fever.

            12-the immuonoglubin which contain five chain in the heavy chain is:
            A-IG g.
            B-IG A.هذا الي انا اخترتو
            C-IG D.
            D-IG E.
            هذي الاختيارات الي جاتني بالتحديد مافيigm والله ما جاني ig m عشان ما تقولو اني اكذب او اني نسيت الاختيار
            13-the foundmental of erythrocyte production is:
            A-ERTHROPIOTEIN.

            14-leucocyte increase incase:
            A-BACTERIAL infection .
            B-chemotherapy.
            C-radiation.
            D-ALL OF ABOVE.
            E-NON OF ABOVE.

            15-the triple granule which is the most aboundent in leucocyte is:
            netrophil.

            16-جاني اسم جهاز حق مايكرو يقلك وظيفتو في الenrobactriace is:
            A-genatic.
            B-biochemecal وهذا الي انا اخترتو
            C-immuno.

            17-IN the serology section we use the following tube :
            plain tube.

            18-which of the following will increase the calcium concentration:
            prolonged application of torniqate.

            19-ferritin measure:
            iron diffeciancy anemia.

            20-the following index sensitive and will measure the totalof iron dff anemia:
            A-TRANSFERRIN.
            B-FERRITEN.
            C-TIBC.
            D-HB CONCENTRATION.
            E-ALL OF ABOVE انا اخترت ذا

            21-TO DIFFRENIATE BETWEEN hemophilia A and B:
            factor assay

            22-the following related to immunassay except:
            IMMUNORECIPENT.

            23-heamophilus influenza cause:
            N.meningitis in infant and cheldren.هذا الي انا اخترتو

            24-the following microorganism will not take the gram stain:
            mycobactrium specius.

            25-the following parasite will not take the gimsa stain:
            A-lishmenai.
            B-TRYPONSOM.
            C-الشستوسوما والله انا اخترت B الترايبنوسوما بس ماني متاكد
            26-plasmodium will transfare by:
            A-MOSIQUTE.انا اخترت ذي
            B-FLY.
            C-ALL OF ABOVE

            27-when the micro invade the body B-cell will secret and it will transfer to :
            plasma cell.

            28-B-plasma cell will immuture by :
            thyroid glandهذا الي انا اخترتو ماني متاكد
            29-T-lymphcyte will immuture by:
            lymph node انا اخترت ذي بس ماني متاكد
            30-glycogene will formation in:
            liver.

            31-the perfect picture of acute leukemia:
            والله جاتني اختيارات غريبه سامحوني نسيتها

            32-wucheria bancrofti will measure by :
            blood and urine.

            33-the infective stage of ankylustoma dudenial is:
            pathogenic larva.

            34-diagnostic stage of ascarias is:
            egg.

            35-which of the following parasite will cause abscess in liver:
            E.hostolytica . هذا الي انا اخترتو بس ترو ماني متاكد

            36-جبلي اسم نظريه بعدين مكتوب turbiduty sample:
            refraction of light هذا الي انا اخترتو جاتني اختيارات كثيره بس نسيتها

            37-حاطط في السؤال اسم وبعد الاسم كاتب مادة الفورمالين يقول انا احسن طريقة في الsedment stool because:
            معليش والله ناسي الاختيارات بس متذكر وحده الي هيmaintain the parasite

            38-the agar of C.diphtheria:
            A-Ss agar.انا اخترت ذي
            B-TCBS.
            C-BLOOD MONITOL AGAR.بس واحد من الشباب قلي ذي
            D-XLD
            ترو ذي نفس الاختيارات الي جاتني والللللله حلفتلكم مو عشان تقولو تيكر كذاب ولا واحد يجي يقلك لا باضيف اختيار من عندي

            39-THE nutrent media is:
            basic media.

            40-the following tests we doing in emergency cases except:
            A-electrolight.
            B-GLUCOSE.
            C-BLOOD GAS.
            D-ALBUMIN. هذا الي انا اخترتو

            41-anti diaurtic hormone will secret from postirior pitutary gland: يعني كانو يقلك اش وظيفة هذا الهرمون:
            A-DECREASE urine volume .وهذا الي انا اخترتو
            B-DECREASE SUGAR VOLUME.
            باقي الاختيارات نسيتها

            42-reteculocyte normal range:
            0.1-0.2%

            43-the meninum of platelate storge :
            5 days.

            44-platelate will store at:
            20-24

            45-psudomonus is :
            A-large mucoid amount .وهذا الي انا اخترتو
            B-SMALL AMOUNT OF MUCOID.
            باقي الاختيارات نسيتها

            46-N.gonorrehea:
            A-GRAM (-)rods only.
            B-OXIDASE (+)ONLY.انا اخترت ذا وشاكك انو تكون الي تحتها الصح
            C-GRAM (-)RODS AND OXIDASE (+)
            باقي اختيار رابع نسيتو

            47-في السؤال ذا جبلي نوع من السيدمونس بس نهايتها بحرف الZ سامحوني نسيت اسمها السؤال يقول THE FOLLOWING CHARACTER IS TRUE:
            A-MOTILE-OXIDASE (+)-LACTOSE FERMENTER-GRAM(-)RODS.هذا الي انا اخترتو ماني متاكد
            ياقي الاختيارت بس عكس

            48-factor of erythropiosis:
            والله ناسي الاختيارات هذا الي انا متذكرو:
            A-NEEDS FOLIC ACIDS AND VITB12.
            B-STIMULUS IN CASE OF HYPOOXYGENEMIA.وعلى ما اعتقد هذا الي انا حطيتو

            49-RETENTION OF URIC ACID:
            KIDNEY.

            50-estimation of acid in heredetary:
            A-AMINO ACID.
            B-FATTY. هذا الي انا اخترتو ماني متاك منو

            51-eneterococci can be diffrentiated from :هذا السؤال عكس الي حطبو دكتور ابو رشاد

            A-S.AUREUS.وهذا الي انا اخترتو
            B-STREPTO.


            52-WILL CAUSE BURSE معليش نسيت السؤال كامل:
            A-STAPH.وهذا الي انا اخترتو ماني متاكد
            B-STREPTO.


            53-THE proper result of indivial:
            when the result be close to the normal value.

            54-reliable method to measure the PH is:
            A-PH meter. هذا الي انا اخترتو
            B-LITMUS paper.
            C-INDICATOR.
            D-NON OF THE ABOVE.

            55-CAUSIN OF WHOOPING CAUGH :
            BORDETELLA PRUTISIS

            هذا كل الي اتذكرتو وسامحوني اذا في تقصير بس في سؤال اخير افراد عصابتي يعرفو بحطو عشان يمكن يجيكم في اختبار الانضمام لعصابتي

            WHICH OF THE FOLLOWING SOFT DRINK WILL GIVE THE GUT:

            A- PEPSI.
            B-7 UP.
            C-7 DOWN.
            D- DEW
            الشاطر يقلي الاجابه

            VII-هذا من شخص آخر :
            موقع مختبرات العرب
            http://www.arabslab.com/vb/showthread.php?t=10961
            هذه أسئلة و لكن الإجابات غير مؤكدة لأنها إجابة طالب :

            بعض اسئلة (أخصائي المختبر) لاخر اختبارين 31-05-2009 ثم 04-06-2009

            1- Short arm chromosome
            a- b b- k c- p d- q

            2- Denaturation of DNA in PCR
            a- 60 b-94 c100 d-56

            3-Taq polymerase used in
            a-ELIZA b-PCR c-Electrophoresis

            4-Sickle cell anemia treat HBAs differentiate from Hbss by
            a-electrophoresis b-ELISA c-solubility test

            5-bands in electrophoresis measure by
            a-by meter b-measure by ruler c-electrophoresis marker ladder

            6-Sickle cell anemia in saudia Arabia distributed in
            a-east b-weast c-south d-middle area e-north

            7-megaloblastic anemia seen in
            a-Gastroctomy b-Folic acid deficiency c-B12 deficiency

            8-E.coli
            a-motile lactose ferment b-non motile non lactose 3- non motile lactose ferment

            9- all the following are true about Salmonella and proteus except
            a-non lactose ferment unmotile b-urease positive c- lactose ferment motile

            10-substance used in Indol test
            a-tryptophane b-phenylalanin c-methyl red

            11-chemical tests commonly use
            a-blood b-Serum c-plasma

            12-Carcinoma in respiratory system associated wit
            a-HBV b- CMV 3-HIV

            13-Strepto pyogenes group A in blood agar show .
            a- α-hemolytic clear zone b- β hemolytic green zone c-gamma hemolytic

            14-mRNA to cDNA by
            a-nested PCR b-RFLP PCR c-RT-PCR d-tRNA

            15- actinomycetes infect
            a-skin b-lung c-intestin

            16) degredation of glucose in anareobic condition
            1-purvate
            2- lactate

            17)small rna virus
            1- picornavirus
            2- adenovirus
            18)heparine tube coilor
            1-green
            2-yello
            19)non motile gram nnegative rod
            1-salmonella
            2-E.COLI
            3-KLEBSIELLA
            20)LDH USED IN
            1-CARDIAC TEST
            2-SKELETAL TEST
            3-LIVER TEST
            4-HAVE WIDESPREAD USE IN CLINICLAL CHEMISTRY TESTS

            21)TARGET CELL VIEW IN
            1-MEGALOBLASTIC ANAEMIA
            2-HBC
            3-AUTOMIMUNE HAEMOLOITIC DISEASE OF NEW BORN
            22) MOST ABUNDANT CELL VIEW IN BLOOD FILM
            1-RED BLOOD CORPSCULAR
            2-PLTS
            3-NEITROPHILE
            4- MONOCYTE

            23)IF YOU USE ACETONE TEST ACID TEST KETONES THE MEDIA SHOULD CONTAIN
            1- GLUCOSE
            2- TRYPTOPHANE


            نسبة انتشار24( Familar Hypercholestremia
            وكانت الأختيارات كالتالي
            1000/1
            500/1
            100/1
            5000/1

            25)the first copy of P.C.R is
            1- 14
            2- 8
            3- 4
            4-32

            26)hepatitis e , c , d is transsmitted by
            1- food contamination
            2- vomiting
            3- fecal-oral
            4-

            27)citrate is considered in acetyle co a
            1-succinyl
            2-malonyl
            3-
            4

            28)P.C.R
            1- polymerase chaine reaction


            1-plasma cel تنشأ من
            2-الخلية التي تنتج اجسام هي
            3-نسبة الخطء في DNA extraction هي
            4-كيف نفرق بين الشخص المصاب بانيميا فقر الدم المنجلي والحامل للمرض
            5-طريقة حسلب MCV
            6-الطريقة الصحيحة لاخذ عينة sputum
            7 - الجلطة اكثر شيوعا في الاوردة ام الشعيرات الدموية ام ام الاوردة
            8-طرق تشخيص الترايكوما
            9-العينة التي ترسل للمختبر باقصي سرعة csf
            10-الخلية التي تفرز mucus في المعدة هي
            11-اورام الامعاء الغليظة تكون اكثر انتشارا في (اي جزء من الامعاء)
            12-بماذا تقاس الband في الالكتروفورسس
            13-اهم الامراض التي تحتاج لتشخيص نخاع العظم
            14-مضاد التخثر الذي لايستخدم في بنك الدم
            15-اهم اسباب cresentation necrotic
            16اهم اسباب انتفاخ الخلية في hypoxia injury
            17-heterophuse antibody تستخدم لتشخيص EBV
            18-ماهو الجزء من الجسم المحتوي +contain lymphocyte germinate center +blue colour
            في مجموعة اسئلة جات عن الاورام مااذكرها لان المصطلحات صعبة شوية
            19-شخص مصاب بالتهاب السحايا كيف يكون شكل البكتريا في العينة
            20-شخص مصاب في lower respiratory tract كيف تكون عدد neutrophils+squamouse cell
            21-لتثبيت عينة سوائل جسم نستخدم
            22-basobasl cell تظهر في الاشخاص التالية عدا
            23-تتميز عائلة enterobacteraces في انها عدا مايلي
            24-يعتمد تقسيم lancefield علي
            25-دورة حياة RBC
            26-تتميز IDA بما يلي عدا انها
            27-نقص فيتامين B12 يسبب
            28-الدودة التي يصعب الكشف عنها في البراز هي HYDATIA.CYST
            29-LISTERIA MONOCYTOGENESSIS GRAM POSITIVE ROD
            30-يختلف البلازما عن السيرم في
            31-الذي يتسبب في نقص MEGAKAROCYTE في نخاع العظم مايلي
            32-السبب في ظهور العينة علي شكل LIPMIC SAMPLE
            33-الوقت اللزم لنمو البكتريا الهوائية
            34-الفحص المستخدم لتفريق بين الSTAPH +STREPT
            35-سؤال عن تحضير البفر
            36-تتميز الE.COLI في انها عدا مايلي
            37-نتيجة تحلل النشا في الامعاء
            38-زيادة افراز الانسولين يسبب
            39-مرض FAMILY HYPERCHOLESTREMIA سببه خلل في جين
            40-يستخدم الCOAGULASE لتفريق بين
            41-العينة الاكثر استخداما في قسم الكيمياء
            42-عند اضافة الكرستال فوليت علي الماكونكي هذا يجعلها تثبط نمو الENTEROCOCCUSE
            43-البكتريا المسببة ل PYLONEPHRITITIUS
            44-انزيم AST يرتفع جدا في الحالة التالية
            45-اختصار ال PCR
            46-شريط الDNA يتكون من طرفين 3+5
            47-اهم الامراض اللي تحتاج لتشخيصها نخاع العظم
            48-في سؤال عن LOCATION TUMUR وجاب عدة اختيارات مااذكرها
            بعض اسئلة (أخصائي المختبر) لاخر اختبارين 04-06-2009
            a- EDTA
            b- sodium citrate
            c- heparin
            c- no anticoagulant
            2)Specific test to diagnose syphilis:
            a- RPR
            b- VDRL
            c- ????
            d- ????

            تعليق


            • #7
              هذه الملخصات لأخونا شادي جزاه الله خيرا هو و من ساعده فيها هذا هو الموقع و شكرا

              هذه الملخصات لأخونا شادي جزاه الله خيرا هو و من ساعده فيها هذا هو الموقع و شكرا :

              الــمــلخصات العــشرة !!! لإجــتـــيـــاز الهـــيــئــة:

              http://www.arabslab.com/vb/showthread.php?t=22685&highlight=%C7%D3%C6%E1%C9+% C7%E1%E5%ED%C6%C9+%C7%E1%CF%DF%CA%E6%D1%E5+%DE%D8% D1%C9+%E3%D8%D1

              تعليق

              يعمل...
              X